Bryant - Course 6. Estate Planning. 9. Trusts Flashcards

1
Q

Module Introduction

An essential technique to be considered in the structure of an estate plan is a “trust.” Because of the flexibility and convenience resulting from properly implementing a trust, trusts have become extremely popular as estate planning techniques.

A trust arrangement is one in which one party (the trustee) holds legal title to property for the benefit of one or more beneficiaries. The beneficiaries are the equitable owners of the trust because they are entitled to the trust property.

The party serving as trustee has a fiduciary obligation to the beneficiaries. As the legal titleholder of the trust property (commonly called the corpus, the principal, or the res), the trustee has certain administrative responsibilities and management authority over the trust assets.

A

The Trusts module, which should take approximately four and a half hours to complete, will explain the different kinds of trusts, their utility, and their tax implications.

Upon completion of this module, you should be able to:
* Name the different kinds of trusts and explain how they work,
* Describe the different kinds of charitable trusts,
* Explain the tax implications of trusts, and
* Describe the conditions under which a trust terminates.

How well did you know this?
1
Not at all
2
3
4
5
Perfectly
2
Q

Module Overview

A trust is an essential component in the structure of an estate plan. Trusts are popular because of the flexibility and convenience with which they can be used as estate planning techniques.

A trust can be as complex or as informal as the individual establishing it. The individual establishing the trust is commonly referred to as the grantor, settlor, or creator.

Responsibility for the tax liability depends on the type of trust created.
* For example, the grantor is taxed on all appreciation of assets within a revocable trust.
* If an irrevocable trust is a grantor trust for income tax purposes, then the grantor, and not the trust entity, is considered the taxpayer.

A

Tax savings can occur if a trust is implemented properly, and tax can be saved in one’s lifetime in the form of income and gift tax savings as well as at death in the form of estate tax savings.

Lastly, it is important to remember that even trusts have a certain duration. Please note, however, that some states have enacted statutes that modify or nullify this rule. The rule against perpetuities will not allow a trust to go on forever. Once the purpose for which the trust was initially created is served, it terminates. But this is not the only reason or cause for termination. There are a host of factors and conditions under which a trust can terminate even before its purpose is served.

To ensure that you have an understanding of trusts, the following lessons will be covered in this module:
* Defective Trust
* Powers of a Trustee
* Types of Trusts
* Income Tax of Trusts

How well did you know this?
1
Not at all
2
3
4
5
Perfectly
3
Q

Section 1 - Fundamentals of Trusts

A trust may be defined as an arrangement in which a trustee holds legal title to property for the benefit of the beneficiaries named by the grantor.

A trustee is entrusted with the entire responsibility of the property as articulated by the grantor. The trustee must distribute and/or accumulate income based upon the trust’s provisions.

Ideally, any trust’s terms and conditions should be written. This reduces the chances of ambiguity and eliminates the possibility of error.

Trusts are used for purposes other than saving estate taxes - even when the estate is not large enough to be subject to tax, trusts are used to ensure that the beneficiaries have assistance in managing and investing funds.

A

A trust is a financial management tool that allows the grantor to coordinate the investment, use, and distribution of property during one’s lifetime and after death.

To ensure that you have an understanding of trust, the following topics will be covered in this lesson:
* Defining a Trust
* Trust Elements

Upon completion of this lesson, you should be able to:
* Define a trust,
* Explain the role of a trustee,
* List the requirements of a trust,
* Explain the conditions of a trust, and
* Explain the advantages of a trust.

How well did you know this?
1
Not at all
2
3
4
5
Perfectly
4
Q

What are the five elements of a trust?

A

Defining a Trust
A trust arrangement is one in which one party, the trustee, holds legal title to property for the benefit of one or more beneficiaries.

There are five elements of a trust:
* grantor,
* trustee,
* corpus (or res),
* terms of the trust, and
* beneficiaries.

How well did you know this?
1
Not at all
2
3
4
5
Perfectly
5
Q

Match the descriptions on the right to the correct corresponding elements on the left.
Beneficiary
Grantor
Trust Terms
Corpus
Trustee
* The written instrument of a trust’s provisions.
* Any person who transfers property to and dictates the terms of a trust.
* A party to whom property is transferred by the grantor and who receives legal title to the property placed in the trust.
* A party that will benefit from creation of the trust and will receive direct or indirect benefit of the use of the trust property and/or income.
* The amount of principal in a trust.

A
  • Trust Terms - The written instrument of a trust’s provisions.
  • Grantor - Any person who transfers property to and dictates the terms of a trust.
  • Trustee - A party to whom property is transferred by the grantor and who receives legal title to the property placed in the trust.
  • Beneficiary - A party that will benefit from creation of the trust and will receive direct or indirect benefit of the use of the trust property and/or income.
  • Corpus - The amount of principal in a trust.
How well did you know this?
1
Not at all
2
3
4
5
Perfectly
6
Q

Describe the role of Trustee

A

A trustee is a party to whom property is transferred by the grantor, who receives legal title to the property placed in the trust, and who generally manages, distributes, and accumulates income and principal as per the terms of a formal written agreement (called a trust instrument) for the benefit of the beneficiaries.

As a fiduciary, the trustee must hold the property, invest it, distribute its income, accumulate income if necessary, and render any services required by law. This is to ensure that the beneficiaries receive the enjoyment and use of the property equitably.
* Someone other than the trustee may be required to pay tax.
* The trustee may be required to file a tax return.

How well did you know this?
1
Not at all
2
3
4
5
Perfectly
7
Q

Define and describe Beneficiary

A

A beneficiary is a party for whose benefit the trust is created and who will receive the direct or indirect benefit of the use of income from and/or principal of the trust property, as follows:
* Income beneficiary: The beneficiary who receives income, generally for life or a fixed period of years or until the occurrence or nonoccurrence of a particular event.
* Remainder person: The ultimate beneficiary of trust property (can be the income beneficiary).

However, depending upon the trust’s provisions, as articulated by the grantor, even a lifetime beneficiary may not automatically receive all income. The grantor may leave this to the discretion of the trustee.

How well did you know this?
1
Not at all
2
3
4
5
Perfectly
8
Q

Define Inter Vivos Trust

A

An inter vivos trust is a trust which takes effect and is funded with assets during the lifetime of the grantor.
* An inter vivos trust may be either revocable or irrevocable.
* On the other hand, a testamentary trust is revocable until the creator dies, at which point it becomes irrevocable.

How well did you know this?
1
Not at all
2
3
4
5
Perfectly
9
Q

What elements are required for the creation of a valid trust?

A

In most states, the following elements are required for the creation of a valid trust:
* There must be specific property. The property must be specifically identified as the property constituting the corpus of the trust.
* There must be one or more ascertainable beneficiaries who will receive equitable ownership of the income through their use and enjoyment of it. In the case of property that is non-income producing, the beneficiary must receive at least the use and enjoyment of the corpus. Though the beneficiaries must be ascertainable, there is no requirement that they are named individually in the trust agreement. Every trust has current and remainder beneficiaries. A common example of trust beneficiaries whose identities are ascertainable but who are not named in the trust itself would be a class of beneficiaries, for example, “the grantor’s grandchildren.”
* There must be a trustee who holds legal title to the property and who administers the property for the benefit of the beneficiaries. Though usually identified in the terms of the trust agreement, many states do not require the trustee’s identity to be established. If the trustee is not identified in the terms of the trust agreement, or if the designated trustee predeceases the grantor, most states permit a trustee to be appointed by the court. Most courts take the position that if it was the intent of the grantor to create a trust, the trust will not fail for having failed to designate a trustee. To avoid having the court appoint a successor trustee, the grantor should name a successor in the trust document.
* There must be an intention to create a trust that is clear from the grantor’s language or actions. Though the use of the words trust, trustee, or trust arrangement may not be required, the language must indicate the clear intention of the grantor to separate legal and equitable titles in the property while having one party serve as fiduciary for the others. It is not enough that a grantor in his wills leaves the property to another with the hope or desire that the recipient creates a trust. Usually, this language evidences no trust arrangement.
* If the trust is of a type that is required to be in writing, then the written requirements must be complied with.
* Finally, most trusts are required to have terms or conditions under which the trust will terminate or fail. If the trust does not have conditions or terms under which it will terminate, so that the trust could conceivably be of infinite duration, then the trust will violate the rule against perpetuities and will fail unless the trust beneficiary is a charity. The rule against perpetuities requires that a trust cannot last longer than the life of a beneficiary who was alive when the trust was created and an additional 21 years and 9 months after the beneficiary has died.

How well did you know this?
1
Not at all
2
3
4
5
Perfectly
10
Q

Describe Professional Management of Assets in a Trust

A

The grantor chooses trustees, and they can be individuals or institutional trustees such as banks or trust companies. Individual and institutional trustees can be named to the same trust since trusts can have multiple trustees.
* An institutional trustee is useful for a client who wants professional management of assets upon death, as well as during lifetime, particularly in the event of mental incapacity.
* Clients desire such an arrangement when they know that the surviving spouse and/or other family members have little professional investment or business experience.
* Using a trust can assure the client that a trustee with professional asset management experience is available to serve in that capacity.

Trustees, whether individual or institutional, have a fiduciary duty to the beneficiaries always to put their interests first.
* Trustees can be sued for a breach of fiduciary duty by the trust beneficiaries if any self-dealing, mismanagement, or wrongdoing occurs.

How well did you know this?
1
Not at all
2
3
4
5
Perfectly
11
Q
A

Structure of Income
Since the income needs of the beneficiary are subject to change, the trustee may be in a better position to determine the actual needs of the beneficiaries than the grantor, who may be deceased.

Income may be accumulated at the trustee’s discretion, depending on the trust provision. Suppose the grantor wants the trustee to have the authority to ascertain the needs of the beneficiaries. In that case, the grantor should allow the trustee to make income distributions “at the discretion of the trustee.”
* Also, remember that from an income tax standpoint, particularly with an irrevocable trust, which is not a grantor trust for income tax purposes, the trust may be in a higher tax bracket than the beneficiaries.
* Therefore, the trustee can manage income tax liability in the other direction by applying the income away from the trust - the higher tax bracket payer - to the beneficiaries who are in lower tax brackets.

Practitioner Advice:
* In a grantor trust, which attributes the income tax liability of the trust assets to the grantor, there is no opportunity to switch the taxpayer for income tax purposes.
* However, suppose the trust is a non-grantor trust for income tax purposes, meaning someone else is responsible for paying the tax. In that case, we need to know whether the trust is a simple trust - all income to beneficiaries, subject to income tax at their rates,
* or a complex trust - income may be accumulated, for which we need to use the DNI calculation to determine the tax liabilities. DNI stands for Distributable Net Income and will be defined under the Tax Treatment of Trusts.

How well did you know this?
1
Not at all
2
3
4
5
Perfectly
12
Q

Practitioner Advice:

Practitioner Advice:
* In a grantor trust, which attributes the income tax liability of the trust assets to the grantor, there is no opportunity to switch the taxpayer for income tax purposes.
* However, suppose the trust is a non-grantor trust for income tax purposes, meaning someone else is responsible for paying the tax. In that case, we need to know whether the trust is a simple trust - all income to beneficiaries, subject to income tax at their rates,
* or a complex trust - income may be accumulated, for which we need to use the DNI calculation to determine the tax liabilities. DNI stands for Distributable Net Income and will be defined under the Tax Treatment of Trusts.

A
How well did you know this?
1
Not at all
2
3
4
5
Perfectly
13
Q

Describe Special Needs Trust

A

If a client wishes to assist a family member who has suffered a disability through an accident or who has been disabled since birth or childhood, a trust arrangement can be used. These trusts are known as supplemental needs trusts, special needs trusts, or craven trusts.

A trust arrangement can provide extra amenities to a disabled person in a way that would not disqualify the disabled beneficiary from receiving public assistance benefits, such as Medicaid/Medicare, Social Security disability benefits, or other forms of supplemental assistance. If a trust is structured correctly, it can achieve all of these objectives.

How well did you know this?
1
Not at all
2
3
4
5
Perfectly
14
Q

Section 1 - Fundamentals of a Trust Summary

Trusts are a popular and effective tool for estate planning.

With the help of a trust, a grantor can leave his or her estate to any beneficiary he or she chooses.

There are different types of trusts, but they can be broadly classified into living and testamentary trusts.

The grantor names or appoints a trustee within the trust document. A trustee is the legal titleholder of the property. There are certain requirements that the trust must meet to be considered valid. A trust makes arrangements for a grantor, keeping his or her specific requirements in mind.

A

In this lesson, we have covered the following:
* A trust is an arrangement in which one party, the trustee, holds legal title to property for the benefit of one or more beneficiaries. In order for a trust to exist, there must be trust property (also known as trust principal, res, or corpus).
* Trust Arrangements can be used for a variety of purposes. These include:
* Professional management of trust assets
* Distribution of income and/or principal to the beneficiary
* Assist family members of the grantor with special needs

How well did you know this?
1
Not at all
2
3
4
5
Perfectly
15
Q

Beneficiaries are equitable owners of the trust in the sense that they are: (Select all that apply)
* Entitled to the enjoyment of the trust property
* Entitled to the income produced by the trust property
* Entitled to hold legal title to the property.
* Expected to pay the trust’s income tax liability

A

Entitled to the enjoyment of the trust property
Entitled to the income produced by the trust property

  • Beneficiaries can be considered to be equitable owners of the trust in the sense that they are to enjoy the property or its income.
  • A trust arrangement is one in which one party (the trustee) holds legal title to property for the benefit of one or more beneficiaries. As a fiduciary, the trustee must hold the property, invest it, distribute its income, pay the trust’s income tax liability, accumulate income, if necessary, and render any services required by law to ensure that the beneficiaries receive the enjoyment and use of the property in an equitable manner
How well did you know this?
1
Not at all
2
3
4
5
Perfectly
16
Q

The advantages of a revocable trust include: (Select all that apply)
* Flexibility
* Easily amendable
* Does not incur estate tax liability
* Does not incur income tax liability

A

Flexibility
Easily amendable

  • The main advantage of a revocable trust is that it provides the grantor with flexibility if the corpus needs to be consumed by the grantor or the grantor’s family.
  • It also provides the grantor with greater ease in amending the trust if the grantor is unhappy with the way in which the trust is operating.
  • The primary disadvantage of a revocable trust is that the assets placed into the corpus of such a trust are included in the gross estate of the grantor and incur both estate tax and income tax liability.
How well did you know this?
1
Not at all
2
3
4
5
Perfectly
17
Q

Trust property better known as which of the following?
* Corpus
* Assets
* Possessions
* Belongings

A

Corpus

  • Trust property is often referred to as corpus or principal.
How well did you know this?
1
Not at all
2
3
4
5
Perfectly
18
Q

Section 2 - Powers of a Trustee

The trust agreement’s terms determine a trustee’s duties and powers. The powers of a trustee can vary from state to state.

A trustee holds legal title to the property and administers the property for the benefit of the beneficiaries. If the trustee is not identified in terms of the trust agreement or if the designated trustee has predeceased the grantor, most states provide for a trustee to be appointed by the court. However, the grantor can name successor trustees. A Grantor can also be a trustee and a beneficiary, which is common in revocable trusts.

A

To ensure that you have an understanding of the powers of a trustee, the following topics will be covered in this lesson:
* General Powers of a Trustee

Upon completion of this lesson, you should be able to:
* List the powers of a trustee

How well did you know this?
1
Not at all
2
3
4
5
Perfectly
19
Q

What are the General Powers of a Trustee?

A

Trustees enjoy certain general powers. These include:
* Power to collect trust property, settle claims, sue, or be sued.
* Power to sell, acquire or manage trust property in a manner that is in the best interests of the trust.
* Power to vote corporate shares.
* Power to borrow money and use the trust corpus as collateral.
* Power to enter into contracts and leases that do not exceed the trust’s duration.
* Power to make payments to a beneficiary of the trust.
* Power to make required divisions and distributions of the trust property.
* Power to receive additional assets into the corpus of the trust.

How well did you know this?
1
Not at all
2
3
4
5
Perfectly
20
Q

What if there are Two or More Trustees?

A

If there are two or more trustees, all trustees must act unanimously.
* Suppose the trustees cannot agree upon a unanimous course of action. In that case, a special hearing will have to be held to determine the effectiveness of the trust and the conditions under which one or more trustees may have to step down or resign.
* On the other hand, if the grantor has designated more than two trustees, the grantor may stipulate that agreement by a majority of the trustees is sufficient for any action under the provisions of the trust.

How well did you know this?
1
Not at all
2
3
4
5
Perfectly
21
Q

Section 2 - Powers of a Trustee Summary

The powers of a trustee differ according to state law.

However, trustees enjoy certain general powers in the state in which they operate. If a trust has more than one trustee, and if the trustees are not in agreement regarding some issues, then a special hearing is called to resolve the issue.

A

In this lesson, we have covered the following:
* Powers of the trustee include the power to collect and sell trust property. They also include the power to borrow money and, while doing so, use the trust corpus as collateral. Trustees also enjoy the power to make payments to a beneficiary of the trust and the power to divide and distribute trust property or receive additional assets into the same.

How well did you know this?
1
Not at all
2
3
4
5
Perfectly
22
Q

Section 3 - Trusts

The kind of trust documents a grantor can use for estate planning varies according to the needs, estate size, the amount of estate tax the grantor is to pay, and the specific identifiable objectives of the client. Just as the tax laws change, so do the types of trusts that are available to meet a specific client objective.

A

To ensure that you have an understanding of types of trusts, the following topic will be covered in this lesson:
* Types of Trusts

Upon completion of this lesson, you should be able to:
* Classify the different types of trusts,
* Explain the special characteristics of different types of trusts,
* Identify the differences between revocable and irrevocable trusts, and
* Identify the differences between living and testamentary trusts.

How well did you know this?
1
Not at all
2
3
4
5
Perfectly
23
Q

Describe Revocable and Irrevocable Trusts

A

Trusts may be characterized as revocable or irrevocable. The grantor can terminate a revocable trust at any point in time.

A revocable trust is considered flexible because the terms of the trust can be amended, altered or revoked by the grantor if the grantor feels that they are not suitable. Revocable trusts also provide the grantor with the flexibility needed if the grantor or the grantor’s family needs to consume the corpus. A revocable trust provides the grantor with comparatively greater ease in amending the trust should the grantor become dissatisfied with how the trust is operating.

An irrevocable trust can be used to hold life insurance policies.
* The primary goal of such a trust is to shift the ownership of the policies from the insured’s generation to a succeeding generation in order to remove the policy proceeds from taxation at the death of the insured and/or spouse.

How well did you know this?
1
Not at all
2
3
4
5
Perfectly
24
Q

Describe a Living Trust

A

A trust that takes effect during the grantor’s lifetime is called a living trust.
* This is also known as an inter vivos trust.
* All revocable trusts are inter vivos trusts.

It can be established for a limited period, last until the occurrence or nonoccurrence of a specific event, or continue after the grantor’s death.

A revocable living trust is one created by the grantor during their lifetime in which the grantor retains the right to revoke the trust, change its terms, or regain possession of the property in the trust.

A revocable trust becomes irrevocable when the grantor dies.

How well did you know this?
1
Not at all
2
3
4
5
Perfectly
25
Q

Describe Revocable Trusts

A

Trusts must be legally funded to exist. The grantor transfers the legal title of property to a trustee, and the grantor and trustee must be legally competent parties. Trusts can control who benefits from the property placed in trust, when property is distributed, and who manages the property. Trusts permit the accumulation of income and provide a choice of situs for the management of multi-state assets.

Revocable trusts can be funded, which avoid probate and ancillary probate for real property held out of state.
* Assets in a funded revocable trust are included in the grantor’s gross estate.

Revocable trusts can also be unfunded, therefore assets not placed within a trust are subject to probate.
* An example of an unfunded trust is a Standby trust.
* This trust is created by a grantor to manage trust assets if the grantor becomes incapacitated, or leaves the country for an extended period of time.
* The grantor as trustee and beneficiary names a successor trustee to manage assets for the grantor in these situations.
* Trusts may be intentionally unfunded to fund the trust in the future. Examples are revocable By-pass trusts and Disclaimer trusts.

Revocable trusts have the advantage of being flexible.

Revocable trusts become irrevocable when the grantor dies, however, this should not be confused with a testamentary trust, which is created by the will at the testator’s death.
* The revocable trust can also become irrevocable if the grantor becomes incapacitated according to the definition of incapacity included in the trust document.
* The grantor can decide to make a revocable trust irrevocable at any time, but gift taxes may result when this change occurs.

A disadvantage of revocable trusts is the costs associated with establishing the trust. Some examples of costs are attorney fees, costs for changing the title of property transferred into the trust, and trustee fees for managing trust assets if desired.
Another disadvantage is that revocable trusts offer no creditor protection since the grantor retains too many rights and powers.

How well did you know this?
1
Not at all
2
3
4
5
Perfectly
26
Q

Describe an irrevocable trust

A

An irrevocable trust can reduce the grantor’s estate tax liability, as long as the grantor retains no incidents of ownership over the property. Also, if the grantor retains no powers over the corpus of the trust that could be construed as ownership, the assets placed into such a trust will escape inclusion in the gross estate of the grantor and will avoid probate.

There are certain exceptions to this rule. These occur when certain types of property are transferred to the irrevocable trust, and the grantor dies within three years of the date of the transfer.

Examples are:
* If the grantor retains a right to income for life, or a right to use or enjoy trust property.
* If the grantor retains a reversionary interest greater than 5%. Only the value of the reversionary interest will be included in the grantor’s estate.
* If the grantor retains a general power of appointment or a testamentary general power of appointment over trust assets.
* If the grantor dies within 3 years of transferring a life insurance policy or retains any incidents of ownership in the policy transferred into the trust.

Other situations that would cause the FMV of irrevocable trust assets to be included in the grantor’s estate are:
* If the decedent as sole trustee could distribute or accumulate trust income, or distribute trust corpus to income beneficiaries.
* If the decedent could change or add new trust beneficiaries.
* If the decedent could change the trustee unless the new trustee is an institutional trustee.
* If the trustee could use trust income or corpus to discharge the grantor’s legal obligations such as child support.

How well did you know this?
1
Not at all
2
3
4
5
Perfectly
27
Q

What is a Pour-Over Trust?

A

Pour-over trusts are funded or unfunded revocable trusts established by a grantor while alive, which hold their assets at death.
* Assets from the decedent’s estate, pensions, life insurance death benefits, and out-of-state property can be “poured over” into this trust at death for asset management purposes.
* This way diverse property interests can be managed in one trust for the benefit of the decedent’s beneficiaries.

How well did you know this?
1
Not at all
2
3
4
5
Perfectly
28
Q

What is a Discretionary Trust?

A
  • In a discretionary trust, the beneficiaries do not have a fixed entitlement or interest in the trust funds.
  • The trustee has the discretion to determine which of the beneficiaries will receive the capital and income of the trust and how much each beneficiary will receive.
  • Generally, such trusts are taxed at the maximum marginal rate (MMR). This rule is, however, subject to certain exceptions.
How well did you know this?
1
Not at all
2
3
4
5
Perfectly
29
Q

What is a Spendthrift Trust?

A

Spendthrift trusts or spendthrift provisions in irrevocable trusts may provide beneficiaries a degree of creditor protection.
* Creditors can reach an individual’s personal funds to satisfy their claims.
* With a revocable or an irrevocable trust, any distributions made to beneficiaries can be accessed by creditors as well.
* Therefore, spendthrift trusts are designed to prevent beneficiaries from receiving distributions from the trust, so creditors cannot access their funds.
* This is accomplished by giving the trustee the power to accumulate income and discretionary authority to distribute to a beneficiary.
* A spendthrift clause in the trust may state that the beneficiary cannot assign, pledge, or promise to give away distributions from the trust to others, or the beneficiary may forfeit their interest in the trust.
* A beneficiary must not be given a general power of appointment or permitted to have access to trust corpus through an ascertainable standard, which would provide funds for his health, education, maintenance, and/or support.

Spendthrift trusts also provide creditor protection if they are established solely for a beneficiary’s supplemental support, not for their general support. This prevents the beneficiary from compelling distributions for support in favor of creditors.

States are responsible for enforcing spendthrift provisions in trusts, which vary from state to state. Most states will uphold spendthrift provisions because the beneficiary is not the owner of the trust assets, and they can only receive discretionary distributions.
* However, if the grantor created a spendthrift trust and named himself beneficiary, this is considered a self-settled trust, which would not receive creditor protection in most states.

How well did you know this?
1
Not at all
2
3
4
5
Perfectly
30
Q

What is a Sprinkling or Spray Trust?

A

A trust that permits the trustee to distribute income or corpus among various beneficiaries according to the needs of the beneficiaries is sometimes referred to as a trust with sprinkling provisions or a sprinkling or spray trust.
* A sprinkling trust can distribute income to beneficiaries, and
* a spray trust can distribute both income and corpus.

Under the terms of such a trust, the trustee has the discretion to determine whether a particular beneficiary needs any income. If such a need is determined, the amount that the beneficiary will receive is also the sole discretion of the trustee.
* Since the beneficiary has no right to receive income annually, the beneficiary does not have a present interest.
* Therefore the grantor will not be able to take an annual exclusion for transfers into the irrevocable trust.

How well did you know this?
1
Not at all
2
3
4
5
Perfectly
31
Q

The primary advantage of a sprinkling trust is __ ____??____ __.
* flexibility
* annual income distributions
* annual exclusion eligibility
* preservation of trust corpus

A

flexibility

  • The primary advantage of a sprinkling trust is the flexibility it provides to the trustee in making income distributions to beneficiaries who may not have a need for income in one year but could have needs in subsequent years.
How well did you know this?
1
Not at all
2
3
4
5
Perfectly
32
Q

Describe the Generation-Skipping Trust

A

The generation-skipping trust can provide income to the grantor’s spouse for the spouse’s lifetime. Upon the death of the spouse, or simultaneous with the income distribution to the spouse, the grantor’s children can receive income for their lives. Upon the deaths of all of the children, the income and principal may pass to the grantor’s grandchild, great-grandchild, etc.

Generation-Skipping Trust
* After Grantor’s Death - The trust provides income for the rest of the grantor’s spouse’s lifetime.
* After Death of the Spouse - The children of the grantor receive income for the rest of their lives.
* After Death of All Children - The grantor’s grandchildren receive the outright title to the property in the corpus.

How well did you know this?
1
Not at all
2
3
4
5
Perfectly
33
Q

Describe Characteristics of Generation-Skipping Trust

A

To be an effective means of saving estate and gift taxes for the grantor, the trust must possess the following characteristics:
* The trust must be irrevocable.
* The trust income may be distributed to the grantor’s spouse and children, but the absolute title to the property must “skip” at least one generation. The ultimate beneficiary of the assets is a skip person.
* The outright title to the trust property must pass to someone separated from the grantor’s generation by at least two generations.
* The generation-skipping transfer tax is only imposed on those transfers deemed direct skips, taxable terminations, or taxable distributions. Thus, the generation-skipping transfer tax is imposed only when the property transfers to the skip person.

Before 1976, this type of trust arrangement was structured primarily to avoid estate or gift tax liability in the child’s estate had the property been transferred directly from the grantor to the grantor’s children. Thus, a direct transfer to the next succeeding generation was avoided, and estate and gift taxes were minimized. However, since 1976, when Congress imposed a generation-skipping tax on transfers of this sort, the attraction of such an arrangement for the extremely wealthy has diminished greatly.

How well did you know this?
1
Not at all
2
3
4
5
Perfectly
34
Q

Describe the Uniform Gifts/Transfers to Minors Act

A

There are a variety of trusts that can be used to benefit minors. The most informal of these are not really trusts but custodial accounts established for the benefit of a minor beneficiary under the Uniform Gifts to Minors Act (UGMA), or the Uniform Transfers to Minors Act (UTMA). UTMA is a modification of UGMA and most states have replaced their Uniform Gifts to Minors Act with the Uniform Transfers to Minors Act.

In this type of fiduciary relationship, the property is placed in an account that bears the name of the minor as the legal titleholder.
* The property is then held by an adult who has “custody” of the property until the minor reaches the age of majority. This person is known as the “custodian.”
* UGMA or UTMA accounts are convenient and relatively inexpensive to administer. They do not have significant administration costs or accounting expenses like more formal trust arrangements.

UGMA provides that an adult, while alive, may make a gift of certain types of property to a minor by having the gift registered in the name of or delivered to the donor or another adult person as custodian for the minor.
* These gifts qualify for the annual exclusion amount.

UTMAs are very similar to UGMAs except that UTMAs are more flexible.
* UGMAs authorize bank accounts and purchase stocks, bonds, and mutual funds.
* UTMAs allow greater use of various investments, including real estate transfers, partnership interests, and oil and gas interests.
* UTMAs also allow for testamentary transfers into the minor’s account.

Not all states recognize both forms of gift-to-minor options.
* The correct format will depend upon the state of residency of the minor.
* UGMAs typically terminate at the age of 18 while UTMA terminates at age 21 or 25, depending on the state.
* Custodians and/or donors may find a UTMA a better alternative than a UGMA because control of the assets is not turned over to an 18-year-old younger adult.

Exam Tip: If the donor to the UGMA/UTMA also is named as custodian, and the donor/custodian dies before the child reaches the age of majority, the value of the custodial account will be included in the donor/custodian’s estate.

How well did you know this?
1
Not at all
2
3
4
5
Perfectly
35
Q

Exam Tip:

Exam Tip:
* If the donor to the UGMA/UTMA also is named as custodian, and the donor/custodian dies before the child reaches the age of majority, the value of the custodial account will be included in the donor/custodian’s estate.

A
How well did you know this?
1
Not at all
2
3
4
5
Perfectly
36
Q

What are 2503(b) Trusts?

A

Special trusts that are structured for the benefit of minors include both 2503(b) and 2503(c) trusts.
* These are trusts that meet the requirements of these Internal Revenue Code sections, respectively, in order to qualify contributions for the gift tax annual exclusion.

A 2503(b) trust can provide a beneficiary with a stream of income when the beneficiary is a minor.

  • The trust must distribute the income to the minor annually or more frequently.
  • All or portions of gifts to such trusts will qualify as gifts of present interest for the income beneficiaries, and thus are eligible for the annual gift tax exclusion.
  • The annual exclusion cannot be used to offset the donor’s gift tax for the present value of the remainder interest passing to any remainder beneficiaries.
  • The corpus of a 2503(b) trust need not be distributed to the beneficiary when the beneficiary reaches the age of majority.
  • The corpus will be excluded from the gross estate of the donor who is not a trustee.
  • The corpus will also be excluded from the gross estate of the income beneficiary if the income interest terminates at the beneficiary’s death.
How well did you know this?
1
Not at all
2
3
4
5
Perfectly
37
Q

Example (2503(b) Trust: Present Interest)

Arbitrarily assuming a 7.0% interest rate, if the trust provides income for life to the donee to be distributed at least annually, and if the donee is age eleven, then based on government valuation tables, the amount of the gift which will be considered as a present interest if $10,000 was put into a Section 2503(b) trust would be $9,724 (the actuarial value of an 11-year-old’s right to income on $10,000 paid annually for life).

A

In some respects, a Section 2503(b) trust is more flexible than either a Section 2503(c) trust or a custodial account because a Section 2503(b) trust does not require distribution of principal and unexpended income at age 21 or sooner.
* Furthermore, a Section 2503(b) trust can last for the beneficiary’s lifetime or for any lesser period of time.
* Also, unlike the case with a Section 2503(c) trust or a custodianship, under a Section 2503(b) trust, the principal does not ever have to be paid over to the income beneficiary.
* Trust principal can go to a different donee specified by the trust’s grantor or a person specified by the income beneficiary.

How well did you know this?
1
Not at all
2
3
4
5
Perfectly
38
Q

What are 2503(c) Trusts?

A

An IRC Section 2503(c) trust is a gift tax tool that enables a grantor to make a gift to a minor in trust and still obtain the annual gift tax exclusion.
* The use of this irrevocable funded trust for gifts to minors eliminates many of the following practical objections to outright gifts:
* Brokers are reluctant to deal in securities owned by minors since minors may disaffirm either a purchase of stock that subsequently falls in value or a sale of stock that later rises in value.
* Property titled in a minor’s name is, to a large extent, “frozen.” It is difficult to sell or exchange that property since a minor’s signature on a real estate deed gives the buyer no assurance of permanent title.
* Guardianship must be used to avoid many of the objections of an outright transfer to a minor, but a guardian must generally post bond and account periodically to a local Orphans’ (sometimes called Surrogate’s or Probate) Court.

A gift to a minor through a Section 2503(c) trust will be considered a gift of a present interest (so the gift will qualify for the annual gift tax exclusion) if the income and principal is available for distribution to or on behalf of the beneficiary at any time prior to the time the beneficiary reaches age 21.
* ** Income that is accumulated in the trust is taxed to the trust and not to the beneficiary**. (Regardless of when a person becomes an adult under state law, the “magic” age is still 21 for Section 2503(c) purposes.)

Unexpended income and principal must be distributable to the beneficiary when that individual reaches age 21. If the beneficiary dies before age 21, accumulated trust income and corpus must go to the minor’s estate or appointee according to a general power of appointment.

Even if the minor is legally unable to exercise a power or to execute a will because the minor is under the legal age, this fact will not cause the transfer to fail to satisfy the above conditions. Furthermore, it is permissible to provide that the trust will continue beyond the donee’s 21st birthday, as long as anytime after reaching age 21, the donee can obtain the property in the trust if he wishes. Thus, the trust property does not have to be forced on a trust beneficiary upon reaching age 21.

How well did you know this?
1
Not at all
2
3
4
5
Perfectly
39
Q

Example (2503(c) Trust Transfers and Taxation)

Larry Smith transfers stock in his closely held corporation to three trusts for his three minor boys. Larry is not the trustee for any of the three trusts. Assuming the trusts qualify under Section 2503(c), the irrevocable stock transfer annually to the trusts will be considered gifts of a present interest. This will allow Larry to obtain the gift tax annual exclusion and therefore minimize or eliminate any gift taxes.

A

Gifts, to the extent they qualify for the annual exclusion (without taking gift splitting into account), reduce the value of Larry’s estate, and the income from any dividends paid on the stock is taxed currently either to the trust, if accumulated, or to the children, if distributed.
* Such trusts are often used to start children on a life insurance program by providing the trusts with sufficient cash each year to pay premiums for the insurance on the boys’ lives.
* Alternatively, trust income could be used to purchase non-necessities or be accumulated and used to help provide non-necessities when the children reach college age.

How well did you know this?
1
Not at all
2
3
4
5
Perfectly
40
Q

Describe Section 2702

A

When a donor makes a gift to a donee that is less than the entire property, the donor retains a partial interest in the property and gifts a partial interest away.
* A “retained interest” is one held by the donor both before and after the transfer to the trust.
* The donor in this case has a retained interest that is greater than zero. That’s because the value of a gift is generally determined by subtracting the value of the donor’s retained interest from the fair market value of the property gifted.

Chapter 14 of the IRC affects the value of partial interest gifts to “applicable family members” that are transferred into new or existing trusts, or the assignment of an interest in an existing trust.

Applicable family members under Section 2702(a) include:
* The spouse of an individual.
* The ancestor or lineal descendant of an individual or that individual’s spouse.
* The brother or sister of an individual and the spouse of any such person.

Section 2702 affects the value of a remainder interest gift to family members in trust when the grantor retains an income interest (a partial interest) in the same trust for a period of time.
* The remainder interest is subject to gift tax when the grantor transfers the assets into the trust, therefore the gift tax value of the remainder interest needs to be determined.

Section 2702(a) provides that all retained interests in trusts that are not “qualified interests” are valued at zero.
* Grantors who make remainder interest gifts to family members in trusts must retain a “qualified payment” or the grantor’s retained interest is valued at zero!

A “qualified payment” is a payment fixed in time and amount.
* Annuity and unitrust payments are qualified payments under Section 2702.
* If a grantor retains an annuity or a unitrust payment, then the gift of the remainder interest is valued as the PV of the remainder interest on the date the property is transferred into the trust.
* However, if the grantor can receive all income from the trust the payment is not a “qualified payment” and the value of the remainder interest gift to family members is based on the entire amount transferred into the trust.

How well did you know this?
1
Not at all
2
3
4
5
Perfectly
41
Q

When does Chapter 14 not apply?

A

Chapter 14 does not apply to:
* Sales to family members
* Gifts to non-related persons
* Incomplete gifts
* A transfer of the entire property to family members
* Partial interests bequeathed to family members
* Remainder interests in trusts if income can only be distributed at the sole discretion of an independent trustee
* Certain property settlement agreements
* Charitable lead trusts
* Personal residence trusts

How well did you know this?
1
Not at all
2
3
4
5
Perfectly
42
Q

Describe Grantor Retained Trust - GRAT, GRUT

A

A grantor retained trust, as its name implies, is an irrevocable trust into which the grantor places assets and retains an income interest for a fixed period of years.
* Principal and any trust appreciation at the end of the specified term will pass to a non-charitable beneficiary, usually a family member, such as a grantor’s child.

The purpose of the GRAT and GRUT is to:
* Transfer property to family in trust at a reduced (or zero) gift tax value.
* Pass appreciation in the GRAT to beneficiaries without incurring additional gift tax.
* Reduce the value of the grantor’s gross estate.

In a grantor retained annuity trust (GRAT), the grantor retains a right to payment of income for a fixed period of years. Annuity payments can be level payments or may increase up to 20% per year.

In a grantor retained unitrust (GRUT), the grantor retains a right to payment of a fixed percentage of the value of the trust property (determined annually) for a fixed period of years. As an example, take a $1 million transfer that retains a 5% income stream. With a GRAT, the income will be a fixed $50,000; with a GRUT, the initial payout is $50,000, with the next year’s distribution to the grantor equal to 5% of the value of the assets within the trust. This payout amount could be more or less than $50,000 depending on how well the invested assets perform. The grantor, not the trust, must pay taxes on the income distributed from a GRAT or a GRUT, but tax payments will further reduce the grantor’s gross estate.

With a GRAT and GRUT, the grantor is essentially making a current gift of the right to trust assets to the remainder beneficiary at a future date. This gives the grantor significant gift tax savings since only the present value of the remainder interest in the trust is taxed, not the current fair market value of the assets transferred into the trust.

GRATs are preferable when using assets that are difficult to value, such as closely-held stock or real estate that is expected to appreciate.
* GRATs should be funded with assets that are likely to outperform the federal Section 7520 rate used to value the grantor’s annuity.
* The lower the Sec. 7520 rate, the easier it is for trust investments to beat that rate.
* Any appreciation that exceeds the annuity payouts will be transferred tax-free to the beneficiaries when the grantor’s income interest ends.
* GRUTs are unaffected by lower interest rates since income distributions are based on variable payments.
* Because the GRUT does not “freeze” the grantor’s interest, as a GRAT does with its fixed annuity payments, it is less effective in passing appreciation to the beneficiaries.

GRATs should be funded when asset values are low, which lowers the gift tax and gives beneficiaries greater upside potential for future appreciation.
* A longer payout term for the grantor or a higher annuity payout for a shorter term results in a lower taxable gift of the remainder interest.

The grantor’s annuity interest can be structured to equal up to 100% of the trust’s value to avoid taxable gifts.
* This is known as “zeroing-out” the GRAT.
* The annuity, typically structured for two or three years, pays the grantor the amount initially transferred to the trust plus a return equal to the Sec. 7520 rate.
* The GRAT must state that the annuity is payable to the grantor’s estate if the grantor dies during the trust term for this gift tax strategy to work.

For example, assume a transfer of $1 million into a 10-year GRAT with a Sec. 7520 rate of 3%. An annuity payment of $117,231 per year will zero out the GRAT. However, if the trust earns an average return of 7%, its value at the end of the 10-year term will be more than $347,000, which will transfer to the beneficiary tax-free.

If the grantor survives the term selected, the beneficiaries would receive the trust property and any appreciation in its value without additional gift taxes imposed on the transfer.
* Furthermore, the trust corpus is removed from the grantor’s estate.
* If the grantor does not survive the specified period selected to receive trust income, then a portion of the trust necessary to yield the annuity or unitrust interest would be included in the grantor’s estate.
* The amount includible is determined by dividing the annual annuity by the Sec. 7520 rate in effect at the grantor’s death. If the result is greater than the trust’s value, then the amount in the trust is included in the estate. If the result is less than the trust’s value, then only the smaller calculated amount is included in the grantor’s estate.

A disadvantage of a GRAT or GRUT is that beneficiaries will receive the grantor’s carryover basis in the trust assets rather than a stepped-up basis, had they inherited the property at the grantor’s death.
* Also, with a GRAT, the fixed annuity must be paid to the grantor even if the trustee has to dip into the trust corpus or has to borrow funds to pay it.
* Grandchildren should not be beneficiaries of this trust since the grantor’s GST exemption cannot be allocated to the trust until after the grantor’s income period ends.
* By that time, the grandchildren’s interest could be much greater than when the trust was established.

Practitioner Advice:
* When these split-interest trusts are created, the value of the remainder interest passes to the non-charitable income beneficiaries.
* This is a** gift that does not qualify for the annual exclusion but does qualify for the $12.92 million gift tax exclusion (2023)**.
* A GRAT may be beneficial in an environment where the federal 7520 rate is low.

How well did you know this?
1
Not at all
2
3
4
5
Perfectly
43
Q

Practitioner Advice:

Practitioner Advice:
* When these split-interest trusts are created, the value of the remainder interest passes to the non-charitable income beneficiaries.
* This is a** gift that does not qualify for the annual exclusion but does qualify for the $12.92 million gift tax exclusion (2023)**.
* A GRAT may be beneficial in an environment where the federal 7520 rate is low.

A
How well did you know this?
1
Not at all
2
3
4
5
Perfectly
44
Q

Describe Grantor Retained Trust - GRIT

A

A grantor retained income trust (GRIT) is another type of split-interest trust.
* A GRIT distributes all of the trust’s income to a grantor for a specified number of years and then distributes the trust’s remainder interest to beneficiaries.

A GRIT was commonly used for income property before the adoption of IRC Section 2702. Previously, a grantor could receive all income from a GRIT and receive favorable gift tax treatment for the remainder interest passing to family beneficiaries. Since 1990 this is no longer the case. Now, under Chapter 14 of the IRC, assets transferred into GRITs are taxed at FMV for gift tax purposes, not at the discounted value of the trust’s remainder interest, when family members are trust beneficiaries.

In contrast, GRATs and GRUTs receive favorable gift tax treatment for remainder interests transferred to family beneficiaries.
* The difference in gift tax treatment can be explained by the type of payments the grantor receives.
* An annuity or unitrust payment in a split-interest trust (GRAT or GRUT) is considered a “qualified payment” under Chapter 14 of the IRC.
* All income distributed annually from a GRIT is not.
* Trusts with “qualified payments” and family member beneficiaries can receive favorable gift tax treatment, while GRITs, which do not have qualified income payments, are taxed on the full value of the assets initially transferred into the trust.

GRITs are now limited to transfers of a personal residence or certain tangible property, such as a painting, in situations where the grantor retains the use of the property during the trust term.
* It is also used for split-interest trusts with non-family members such as friends, life partners, nieces, and nephews, which retains its favorable gift tax treatment.

A grantor who establishes a GRIT and dies before the term of years has ended will have the full FMV of the trust corpus included in the gross estate. If the grantor is only entitled to a portion of the trust income (30%) then only 30% of the value of the trust is included in the grantor’s estate.

45
Q

Describe Qualified Personal Residence Trust

A

A qualified personal residence trust (QPRT) is an irrevocable trust that holds a person’s residence, allowing couples or individuals to live in the house rent-free for a specified period of time.
At the end of the term, the home will pass gift-tax free to the trust beneficiaries.
* An advantage of this technique is that the transfer of the home into the trust is taxed at the present value of the home’s remainder interest, rather than at its fair market value.
* The gift tax can be reduced because a QPRT is not subject to the unfavorable gift tax provisions found in Chapter 14 of the IRC.
* The gift tax is further reduced for the grantor’s right to have the residence revert to his estate if he does not survive the selected term.
* However, if the grantor dies before the term is completed, then the fair market value of the home is included in the grantor’s estate.

A QPRT allows the grantor to “freeze” the value of the home when it is placed in the trust, and pass any future appreciation on to trust beneficiaries.
* But if the beneficiaries intend to sell the home in the future, it would be better to keep the residence in the grantor’s estate to receive a stepped-up basis at death.
* Otherwise, a beneficiary’s basis is the grantor’s original basis increased by a portion of any gift taxes paid.

A person can transfer a personal residence into a QPRT and establish another QPRT to hold an additional home.
* It is possible to transfer a rental vacation residence to a QPRT if the owner occupies the home for the greater of 14 days or 10% of the number of days it is rented out during the taxable year.
* QPRTs can be structured as grantor trusts with income tax deductions taken for mortgage interest payments and property taxes paid.
* If the grantor wishes to continue living in the home beyond the term of years selected, the grantor could lease the home back at fair rental value.
* Payment of rent to the trust further reduces the grantor’s gross estate and makes additional tax-free gifts to the trust beneficiaries.
* Moreover, rental payments do not generate taxable income for the trust or the trust beneficiaries.
* The grantor can decide to sell the home before the term ends, but the trust would immediately terminate.
* Instead, the grantor could transfer the property into a GRAT which can hold real estate, to preserve some tax advantages. One such advantage is using the original value assigned to the children’s remainder interest in the QPRT when initially funding the GRAT.

46
Q

When are GRAT, GRUT, or QPRT trusts useful?

A
  • A GRAT, GRUT, or QPRT is particularly useful when the client is single and has a substantial estate upon which federal estate taxes are certain to be paid. Wealthy widows or widowers, divorced individuals, or other unmarried persons can use such a trust as a “marital deduction substitute.”
  • A married couple with an estate above the couple’s combined unified credit equivalent can use a GRAT, GRUT, or QPRT to eliminate or reduce taxes on the death of the second spouse to die. The larger and more rapidly appreciating these estates are, the more effective such a trust would be.
  • A GRAT, GRUT, or QPRT is effective where the income-producing property is located in more than one state and unification and probate savings are desired. The GRAT, GRUT, or QPRT would serve to transfer ownership in a manner that would avoid ancillary administration.
  • A GRIT is a useful technique when a client wants to purchase certain tangible assets such as a work of art, retain the right to display it in his or her own home, and have it pass to a specified person immediately and without probate at death. (However, if the grantor is unable to establish the value of the retained interest through comparable rentals, the gift of the transferred remainder will equal 100% of the value of the transferred property).
  • A GRAT, GRUT, or QPRT can be an alternative to (or be used in conjunction with) a recapitalization or other freezing technique with the added advantages of gift tax leverage and possible estate tax savings.
  • When there is a high probability that the client will outlive the trust term that is needed to obtain a low present value gift to the remainder person. When the client has assets so substantial that a significant portion can be committed to a remainder person without compromising his financial security, and when the client has a high-risk tolerance and a strong incentive to achieve gift and estate tax savings (rather than taking the safer but more costly approach of making an immediate gift).
47
Q

Section 3 - Trusts Summary

Trusts can be classified into revocable, irrevocable, and living and testamentary trusts. There are different kinds of trusts with different characteristics. These trusts are formulated to suit the specific needs of their creators- the grantors. The trusts discussed in this lesson include Living, Testamentary, Revocable, Irrevocable, Pour-over, GRAT, GRUT, Discretionary, Spendthrift, Sprinkling or Spray, Generation-skipping, UGMA, UTMA, 2503(b) and 2503(c).

In this lesson, we have covered the following:
* Living Trust takes effect during the grantor’s lifetime.
* Testamentary Trust takes effect upon the death of the grantor. Testamentary trusts may be a part of a decedent’s last will and testament and are always in writing.
* Revocable Trusts are flexible and easily amendable.
* Irrevocable Trusts as the name suggests, are comparatively more rigid but have the advantage of reducing the grantor’s estate tax liability.
* Pour-over Trusts decedent’s assets from the estate, pension assets, life insurance proceeds and out-of-state property are transferred into this existing trust at death, for asset management purposes.
* GRAT and GRUT are trusts into which the Grantor makes an irrevocable transfer yet retains the right to income from the property for a period of time. The income from the GRAT is fixed while the income from the GRUT is variable.
* QPRT is an irrevocable trust that holds a personal residence and permits a person to live there for a term of years. Transferring a home into a QPRT reduces the gift tax value to the PV of the home’s remainder interest.

A
  • Discretionary Trust -vests the distribution of income or corpus to beneficiaries according to the trustee’s discretion.
  • Spendthrift Trust: a discretionary trust that affords some creditor protection to beneficiaries.
  • Sprinkling or Spray Trust allows the trustee to distribute income or corpus among various beneficiaries, based upon the needs of the beneficiaries.
  • Generation-skipping Trust is designed to ultimately transfer assets to skip people and provide for interim generations prior to the distribution. Properly structured, no or minimal GST tax will be required when the assets pass to the skip person.
  • UGMA is a custodial account, which is used for the benefit of minor children.
  • UTMA is similar to a UGMA, but also allows transfers of cash and other property to be placed into the minor’s custodial account.
  • 2503(b) Trust provides a beneficiary with a stream of income during the trust term and also qualifies for the annual gift tax exemption.
  • 2503(c) Trust allows an annual gift tax exclusion for transfers to the trust with the minor having a right to withdraw accumulated income and principal at age 21.

Exam Tip: Keep the unique planning opportunities offered by each of the testable trusts on the CFP® Exam top of mind. Think to yourself, “How can this specific trust maximize the client’s potential to achieve their stated goals?”

48
Q

Exam Tip:

Exam Tip: Keep the unique planning opportunities offered by each of the testable trusts on the CFP® Exam top of mind. Think to yourself, “How can this specific trust maximize the client’s potential to achieve their stated goals?”

A
49
Q

Mark places his assets into the corpus of a revocable trust. He will not escape estate tax liability or income tax liability. There are, however, some advantages and some disadvantages to placing property in a revocable trust. Choose the advantages associated with a revocable trust from the options given below: (Select all that apply)
* Capable of being flexible and terms of the trust can be altered, amended or revoked.
* The grantor can revoke the trust entirely if he realizes that it does not fulfill his larger estate planning objectives.
* In an event of the grantor’s requirement, he can amend it so that it can be made available for him or his family.
* Assets placed in the corpus of such a trust are included in the gross estate of grantor for estate tax liability.
* The grantor bears the income tax liability for the assets placed in the revocable trust.

A

Capable of being flexible and terms of the trust can be altered, amended or revoked.
The grantor can revoke the trust entirely if he realizes that it does not fulfill his larger estate planning objectives.

  • The primary disadvantage of a revocable trust is that the assets placed into the corpus of such a trust are included in the gross estate of the grantor and do not escape estate tax liability or income tax liability.
50
Q

Beth is a five-year-old child. She inherits a trust property that is structured so that she has the right to claim a maximum amount under a five and five power that is 5% of the value of the corpus. Choose the type of trust with such a provision:
* Crummey trust
* Revocable trust
* Irrevocable trust
* Annuity trust

A

Crummey trust

  • If a trust is properly structured a minor beneficiary can demand the lesser of the amount transferred annually to the trust or the maximum amount allowed under a five and five power which works out to be the greater of $5,000 or 5% of the value of the corpus.
  • In such a case the minor beneficiary has a right, better known as a Crummey right.
  • Such a trust is termed a Crummey trust.
51
Q

Match the trust with the correct description:
Sprinkling or Spray Trust
Generation-Skipping Trust
Support Trust
Discretionary Trust
* A trust that benefits several generations within the same family. It is established primarily to take advantage of estate and gift tax savings for the grantor.
* A trust that permits the trustee to distribute income or corpus among various beneficiaries.
* A trust that vests the distribution of income or corpus to beneficiaries.
* A trust that is established solely for the purpose of providing income to a beneficiary in discharge of the grantor’s obligation of support to the beneficiary.

A
  • Generation-Skipping Trust - A trust that benefits several generations within the same family. It is established primarily to take advantage of estate and gift tax savings for the grantor.
  • Sprinkling or Spray Trust - A trust that permits the trustee to distribute income or corpus among various beneficiaries.
  • Discretionary Trust - A trust that vests the distribution of income or corpus to beneficiaries.
  • Support Trust - A trust that is established solely for the purpose of providing income to a beneficiary in discharge of the grantor’s obligation of support to the beneficiary.
52
Q

Section 4 - Other Types of Trusts

Many other types of trusts serve the needs of clients.

If a client wishes to reduce potential estate tax liability, this objective can be achieved by utilizing various combinations of trusts.

If a client wants to provide that the income from a trust be distributed to specified beneficiaries with the remainder being distributed to a charity, a trust can accomplish this purpose.

Suppose the client wishes to have professional management of the assets and knows that the surviving spouse and other family members have little professional investment or business experience. In that case, using a trust can assure the client that someone with professional asset management experience can serve in that capacity.

If the client wants to provide income to a specified beneficiary but knows that the beneficiary cannot control the money, the client can structure the trust in such a way that the beneficiary cannot transfer the income until it is actually received.

A

Moreover, if the client needs some of that money to continue his or her current lifestyle, he or she can set up the trust in such a way that it pays a monthly or yearly income, even if the tax has already been written off.

Trust arrangements can be used for a variety of purposes.

To ensure that you have an understanding of other kinds of trusts, the following topics will be covered in this lesson:
* Trusts that benefit charities
* Trusts that benefit income and families
* Trusts that avoid probate
* Trusts that benefit taxes and deductions

Upon completion of this lesson, you should be able to:
* Outline the different kinds of charitable trusts,
* Describe the characteristics of charitable trusts,
* Explain how and which trusts avoid probate,
* Explain how trusts benefit spouses and other family members, and
* Describe how trusts minimize estate and income tax liabilities.

53
Q

Describe the Charitable Lead Trust

A

Trusts can be used to benefit a qualified charity. A charitable lead trust (CLT) pays a fixed income stream to a qualified charity for a period of years, usually not more than 20. At the expiration of the lead period, the remainder interest passes to one or more noncharitable beneficiaries.

The charitable lead trust can provide the grantor with either income or estate tax savings, depending on when the use of such a technique is employed.
* The grantor can obtain an immediate income tax deduction for a charitable lead trust established during the grantor’s lifetime when the grantor is considered the owner of the trust income and if the remainder interest will revert back to the donor or the donor’s spouse.
* The deduction is measured by the present value of the income stream passing to the charity for the stated period.
* However, if the grantor takes an income tax deduction for the present value of the income to the charity, the grantor must include the income earned by the trust on the grantor’s annual tax return.
* There is no gift tax on the portion going to charity; however, the remainder benefit is usually noncharitable.
* Note: this remainder interest is a taxable gift if given to someone other than the donor or the donor’s spouse - no annual exclusion would apply.

If the charitable lead trust is structured to take effect upon the grantor’s death, the grantor’s estate will receive a charitable deduction for the present value of the income stream passing to the charity.

54
Q

How long does a charitable lead trust pay a fixed income stream to a qualified charity?
* 10 years
* 20 years
* 5 years
* 15 years

A

20 years

  • A charitable lead trust pays a fixed income stream to a qualified charity for a period of years, usually not exceeding 20 years.
55
Q

Describe Charitable Remainder Annuity Trust (CRAT)

A

A charitable remainder annuity trust (CRAT) is designed to permit a fixed amount to pay annually to a noncharitable beneficiary with the remainder going to charity.

Using a basic charitable remainder annuity trust, the donor transfers money or securities to a trust, preferably low basis stock, since the charity can sell this stock without incurring any capital gains, and reinvest the proceeds to pay the donor a higher fixed dollar amount each year for life.
* If the income of the trust is insufficient to meet the required annual payment, the difference is paid from capital gains or principal.
* If the income is greater than the amount required in any given year, the excess income is reinvested in the trust.

The income tax deduction is computed in the year funds are irrevocably placed in trust and is measured by the present value of the charity’s right to receive the trust assets upon the income beneficiary’s death (or at the end of the term of years).

The value of the remainder interest is determined by a combination of the term of the trust or the beneficiary’s age, the amount payable to the beneficiary, and the appropriate monthly applicable federal rate.

In order to qualify for income (and estate and gift) tax deductions, a charitable remainder annuity trust must meet a number of tests.

56
Q

What tests must a charitable remainder annuity trust meet in order to qualify for income (and estate and gift) tax deductions?

A

In order to qualify for income (and estate and gift) tax deductions, a charitable remainder annuity trust must meet a number of tests.
* A fixed amount or fixed percentage of the initial value of the trust must be payable to the noncharitable beneficiary.
* The annuity percentage must not be less than 5% nor more than 50% of the initial FMV of all the transferred property.
* The specified amount must be paid at least annually to the noncharitable beneficiary out of income and/or principal.
* The trust must be irrevocable and not subject to a power by either the donor, the trustee, or the beneficiary to invade, alter, or amend, the trust.
* The trust must be for the benefit of a named individual or individuals who must be living at the time of the transfer in trust, and their interests must consist of either a life estate or a term of years not exceeding 20 years.
* The entire remainder must go to charity.
* The value of the remainder must equal at least 10% of the assets transferred to the trust.

If all the necessary tests are met, the donor of a charitable remainder annuity trust will be entitled to an income tax deduction equal to the value of the remainder interest assuming his contribution base is sufficient to utilize the full amount of the deduction.

57
Q

Example (CRAT: Calculating Charitable Deductions)

A client is considering transferring $100,000 to a trust that will pay him $5,000 a year for life, with a remainder to charity.
* How will his charitable contribution be determined?
* The client’s deduction would depend on his age at his or her nearest birthday.
* In the illustration below (arbitrarily assuming a 6% Section 7520 rate), a 55-year–old donor would receive a $40,771 deduction.
* A 65-year-old would receive a $51,424 deduction for the same contribution.

Charitable Remainder Annuity Trust - One Life
Value of Property $100,000
Annuity Payment $5,000
Sec. 7520 Rate 6.0%
Age 55
Frequency of Payments Annual
Payments End of Period
Annuity Factor (age 55, 6.0%) 11.8459
Annuity Adjustment Factor (annual, 6.0%) 1.0000
Annuity Value ($5,000 x 1 x 11.8459 x 1.0000) $59,229
Charitable Contribution ($100,000 - $59,229) $40,771

A
58
Q

Describe Charitable Remainder Unitrust (CRUT)

A

A charitable remainder unitrust (CRUT), like a charitable remainder annuity trust, is designed to permit a periodic sum payment to a noncharitable beneficiary with a remainder to charity.
* The key distinction is in how the periodic sum is computed.
* It is also important to note that no additional contributions may be made to a CRAT once created and funded.
* However, future contributions may be made to CRUTs.

For example, a donor irrevocably transfers money or securities to a trustee. In return, the trustee agrees to pay the donor (or other beneficiary) a unitrust amount from the property for life. The donor also requires that if he predeceases his spouse, she in turn will receive a unitrust amount from the donated property for life. The donor will make payment based on a fixed percentage of the fair market value of the assets placed in trust. The assets will be revalued each year.

In order to qualify for income, gift, and estate tax deductions, the structure of a charitable remainder unitrust must conform to guidelines.

59
Q

List the 6 CRUT Conforming Guidelines

A

CRUT Conforming Guidelines
* A fixed percentage of the net FMV of the principal, revalued annually, must be payable to the noncharitable beneficiary.
* The percentage payable to the noncharitable beneficiary must not be less than 5% nor more than 50% of the annual value.
* The unitrust may provide that the noncharitable beneficiary can receive the lesser of: (a) the specified fixed percentage, or (b) the trust income for the year, plus any excess trust income to the extent of any deficiency in the prior years.
* The noncharitable income beneficiaries must be living at the time of transfer in into the trust, and their interests must be for a life estate or a term of years not exceeding 20 years.
* The entire remainder must go to charity.
* The value of the remainder must equal at least 10% of the assets transferred into the trust.

If allowed at all, an income tax deduction is permitted in the year funds are irrevocably placed in trust.
* The deduction is measured by the present value at the date of the gift of the charity’s right to eventually receive the unitrust’s assets, subject to percentage limitation.
* The portion of the deduction disallowed may generally be carried forward for five years.

60
Q

Example (CRUT: Tax Deduction Calculation)

A 67-year-old donor places $250,000 in a charitable remainder unitrust, retaining a payment equal to 9% of the trust’s annual value, with payments to be made quarterly at the end of each period. What is the total income tax deduction available to the donor? Assuming the use of a 6% Section 7520 rate, the donor’s deduction would be $77,985. This computation is illustrated below.

CHARITABLE REMAINDER UNITRUST - ONE LIFE
Value of Property $250,000
Unitrust Payout Rate 9.0%
Sec. 7520 Rate 6.0%
Age 67
Frequency of Payments Quarterly
Number of Months Until First Payment 3
Adjusted Payout Rate Factor (annual, 3 months, 6.0%) 0.964365
Adjusted Payout Rate (9.0% x 0.964365) 8.679%
Unitrust Remainder Factor (age 67, 8.6%) 0.31464
Unitrust Remainder Factor (age 67, 8.8%) 0.30780
Difference [0.31464 - 0.30780] 0.00684
Interpolation Adjustment [0.00684 x ((8.679% - 8.6%) / (8.8% - 8.6%))] 0.00270
Interpolated Remainder Factor [0.31464 - 0.00270] 0.31194
Charitable Contribution [$250,000 x .31194] $77,985

A

Practitioner Advice:
* With charitable remainder trusts, the income must be paid out for a term certain not to exceed 20 years, or the lifetime of the noncharitable beneficiary.
* Additionally, for the CRT to qualify as a CRT, each time a contribution is made into the trust, whether once with the annuity trust, or with each transfer into the unitrust, at the time of transfer the calculation must show that at least 10% of the value of the assets contributed on the day of transfer will go to the charity.

61
Q

Practitioner Advice:

Practitioner Advice:
* With charitable remainder trusts, the income must be paid out for a term certain not to exceed 20 years, or the lifetime of the noncharitable beneficiary.
* Additionally, for the CRT to qualify as a CRT, each time a contribution is made into the trust, whether once with the annuity trust, or with each transfer into the unitrust, at the time of transfer the calculation must show that at least 10% of the value of the assets contributed on the day of transfer will go to the charity.

A
62
Q

Describe Irrevocable Life Insurance Trust (ILIT)

A

An irrevocable life insurance trust (ILIT) is a vehicle for holding life insurance policies.
* The primary goal of such a trust is to remove the death benefit proceeds of the life insurance policy(ies) from the estate of the grantor and the grantor’s spouse and shift the ownership of the policies to a lower generation.
* This effectively removes the policy proceeds from taxation at the death of the insured and/or his spouse.
* In that way, the family can have a fund of cash which can be used to loan money to or purchase assets from a decedent’s estate, thereby creating liquidity in the estate for payment of death taxes, without the fund’s causing additional death taxes at the death of the insured.

  • Where the life insurance continues to require payment of premiums after the initial gift, a typical irrevocable life insurance trust plan will call for the insured to make annual gifts to the trust to cover the premium payments.
  • In order to make those gifts qualify as a “present interest gift” and therefore come within the annual exclusion, the trust gives a “Crummey” withdrawal right to the beneficiaries of the trust (usually the children, grandchildren, spouse, etc.).
  • Note: concerning Crummey rights-it qualifies an otherwise future interest transfer as a present interest transfer.
  • The name “Crummey trust” comes from the name of a party to a lawsuit, Crummey v. Comm. (9th cir. 1968).
63
Q

Describe Use of Life Insurance Trust

A
  • An irrevocable life insurance trust (ILIT) can be used to good advantage whenever an individual (or a couple) faces a death tax in his or her generation.
  • An ILIT can provide the decedent’s estate with liquidity for payment of all death taxes with existing life insurance, or with insurance, the trust intends to purchase, without subjecting the proceeds themselves to depletion by estate taxes.
  • To provide the estate with liquidity, the trustee is given discretionary powers to lend funds from the insurance proceeds to the estate or to buy assets directly from the estate to transfer into the trust for professional management. The loan should be secured and structured as an “arms-length” transaction, with prevailing market interest rates applied. Property sold to the trust should be valued at its fair market value.
  • Gifts of whole life insurance policies to the ILIT are generally valued for gift tax purposes at the “interpolated terminal reserve value,” which is usually close to the cash surrender value for a reasonably healthy individual. The face value for the policy is often many times greater than this gift tax value.
  • In such cases, making gifts of life insurance policies is a way to “leverage” the donor’s annual gift tax exclusions, applicable credit, and/or generation-skipping exclusion, by making gifts of policies that have a low current “gift value” in relation to their death benefit value at the insured’s death.
  • Life insurance trusts can be funded or unfunded trusts.
  • An unfunded ILIT means that the trust only contains a life insurance policy that the owner has transferred in.
  • Since life insurance premiums need to be paid each year, the owner must transfer money for these payments into the trust each year, so that the trustee has sufficient funds to pay the annual premiums.
  • These transfers into the trust are subject to gift taxes, therefore the grantor wants to use annual exclusions for each trust beneficiary to offset these taxes.
  • For this reason, the grantor will include Crummey provisions into the ILIT, which gives the beneficiaries a present interest rather than a future interest in the trust, due to the beneficiaries’ annual rights of withdrawal.
  • A funded ILIT means that the grantor has transferred a life insurance policy plus income-producing property into the trust, which is used to pay the premium payments.
  • The grantor does not need to give beneficiaries a Crummey power in this trust, since beneficiaries do not need to have a present interest in the trust.
  • The reason is that the grantor will not make annual transfers into the trust to pay for the policy’s premiums, since the income generated from the assets in the trust will be used to pay for the premiums instead.
64
Q

Which Trusts Avoid Probate?

A

All trusts avoid probate if the trust is funded with assets prior to the death of the grantor.

  • **Joint tenancy is applicable to all property types except retirement plans and individual retirement accounts (IRAs)
  • Beneficiary designations can be used for life insurance policies, retirement plans, and individual retirement accounts**.
  • Revocable trusts can be utilized with all types of property. The revocable living trust avoids probate, as well as possible gift tax consequences.
65
Q

What is a Totten trust?

A

A Totten trust is not a traditional revocable or irrevocable trust.
* It is a type of revocable bank account or money market account that avoids probate.
* The checking or savings account owner is considered the “trustee” who makes deposits for a named beneficiary.
* Since the account owner has full control over the money in the account until death, it is included in their gross estate at death.
* The account then passes automatically to the beneficiary as a payable on death (POD) transfer, which avoids probate in the owner’s estate.

66
Q

Describe Marital Trusts

A

The marital/non-marital trust (also commonly referred to as the “A-B trust” or, more currently, the “A-B-Q” trust) is an arrangement designed to give the surviving spouse full use of the family’s economic wealth, while at the same time minimizing, to the extent possible, the total federal estate tax payable at the deaths of both spouses.

Married couples can usually eliminate federal estate taxes entirely at the first spouse’s death through a carefully considered plan capitalizing on the marital deduction coupled with the unified credit.
* The marital deduction is a deduction for gift or estate tax purposes for property passing to (or in a qualifying trust, for) a spouse.
* The applicable credit is a credit provided to each citizen or resident of the United States, which can be applied against either gift taxes or estate taxes.

The goal typically is to avoid “overqualification” of the estate for the marital deduction because of the “underutilization” of the applicable credit in the estate of the first spouse to die.

67
Q

Describe the Bypass Trust

A
  • The B-Trust or bypass nonmarital (family) trust is designed to receive property that is not allocated to the power of appointment trust, the estate trust, or the QTIP trust.
  • An amount equal to the exemption equivalent amount ($12,920,000 in 2023) is placed in this trust.
  • The trustee may be directed to distribute the net income of this trust to the surviving spouse during his lifetime, or the income may be accumulated or directed to other persons in order to reduce the overall income tax effect on the family.
  • If drafted properly, the trust can also allow the trustees to distribute principal to the surviving spouse during her life without having the balance of the principal at her death included in the surviving spouse’s estate.
  • Because the life estate terminates at the death of the surviving spouse, the surviving spouse did not create the life estate, and the surviving spouse has no “general” power of appointment over the trust property, there will be no transfer of property that is includable in the estate of the surviving spouse.
  • Thus, this trust “bypasses” the survivor’s taxable estate and is therefore often called a Credit Equivalent Bypass Trust (CEBT).
  • The surviving spouse can be given a “limited” or “special” power of appointment under this family trust. For example, a surviving spouse might be given the right to appoint, at his or her death, all or any part of the assets in the family trust to a limited class of beneficiaries, such as the children of the grantor. This means that the surviving spouse can distribute the assets in this trust only to the specified children, but can do so in any proportion the spouse desires. Unlike a general power of appointment, this limited power of appointment does not cause the trust assets to be included in the surviving spouse’s estate.
  • A surviving spouse can utilize the unused portion of a decedent spouse’s unified credit. The deceased spouse’s unused exclusion amount (DSUEA) is added to a surviving spouse’s basic exclusion amount to reduce gift and/or estate taxes on their combined estates. If a surviving spouse had more than one deceased spouse, then only the most recent spouse’s DSUEA can be used.
68
Q

Example (DSUEA)

Suppose Sean and Samantha were married and Sean died in 2023. Sean had a taxable estate of $2 million and had adjusted taxable gifts of $1 million. His $3 million taxable estate is not subject to estate taxes since Sean’s unified credit of $5,113,800 shelters an estate of up to $12,920,000. The $9,920,000 of his unused lifetime exclusion amount is transferred to Samantha, who now has a lifetime exclusion amount of $22,840,000. (her $12,920,000 basic exclusion plus Sean’s unused exclusion of $9,920,000) Samantha can use this $22,840,000 exclusion against taxable gifts made in her lifetime and/or apply it towards her taxable estate.

A
69
Q

How do you take advantage of the DSUEA?

A

To take advantage of the DSUEA, an executor must take the following actions on the decedent’s estate tax return, Form 706.
* File a timely return or file timely extensions
* Compute the DSUEA
* Make an irrevocable election to utilize the DSUEA

70
Q

For estate planning, when would a By-Pass trust still be useful?

A

For estate planning purposes, the By-Pass trust may still be useful because:
* The appreciation on assets transferred to the trust escape estate taxation in the surviving spouse’s estate.
* The decedent can control who will receive the assets in the trust by selecting the beneficiaries. If assets are given outright to the surviving spouse and the spouse remarries, the assets may not be used as the decedent spouse intended.
* The decedent’s GST exemption is not portable, therefore the decedent’s GST exemption could be allocated to a By-Pass trust if skip persons are trust beneficiaries.
* Couples who live in states that impose an estate tax may use the trust and other techniques to reduce or avoid paying state estate taxes.

71
Q

What are the drawbacks of the bypass trust?

A

The drawbacks:
* Assets in the trust do not receive a step-up in basis, yet assets bequeathed directly to the surviving spouse are stepped up to FMV.
* The value of the assets transferred to the bypass trust will reduce the amount of the decedent’s unused exemption that is available to the surviving spouse.

72
Q

Describe QTIP Trusts

A
  • For married couples with significant taxable estates, a Bypass trust is created first for each spouse with assets equal to the tax year’s exemption equivalent amount. That amount is $12,920,000 in 2023.
  • Any excess amounts can be given directly to the surviving spouse or transferred into other marital trusts, so that the decedent’s estate will receive a marital deduction to offset a potential estate tax liability.

Examples of marital trusts that qualify for the marital deduction are general power of appointment trusts (A trusts) and Estate trusts.

QTIP trusts can be subject to the marital deduction in the decedent’s estate if the executor qualifies the terminable interest property “QTIP” for the marital deduction.

  • Placing assets in this QTIP trust can result in zero estate tax at the first spouse’s death to the extent that the trustee or executor elects to take the marital deduction on Form 706, the estate tax form.
  • All of the income of this trust must be paid, at least annually, to the surviving spouse, and the spouse can have a 5 x 5 power over the trust corpus.
  • No provision for invasions of the trust can be made for anyone other than the surviving spouse, or the marital deduction will be lost. Likewise, it would be lost if there were any condition (for example, remarriage) or power in anyone else which could prevent the surviving spouse from receiving all the trust income for life.
  • If the trustee has elected to take a marital deduction at the first spouse’s death for assets going into that trust (usually 100%), the QTIP (Q) trust will be included in the surviving spouse’s estate. The value of the assets remaining in the QTIP will be included in the surviving spouse’s estate. Any assets remaining in this trust at the survivor’s death will be distributed as the grantor decided.
  • If generation-skipping is planned and if some of the grantor-decedent’s exemption from the generation-skipping transfer (GST) tax is still available, the executor will make a “reverse Q-TIP” election, which will still allow a marital deduction in the decedent’s estate, and will allocate the remaining GST exemption to the trust.

Provisions are usually made for the QTIP trust to be divided into two trusts:
* Trust Q-E: Will contain property to which some of the GST exemption will be allocated.
* Trust Q-N: Will not be exempt from the GST tax and, therefore, will often have different terms than the trust Q-E.

73
Q

Describe QDOT Trusts

A
  • An estate may claim an unlimited deduction for the value of all property that passes from a decedent to the surviving spouse upon death. However, no marital deduction is allowed for the estate of a U.S. resident who leaves property outright to a surviving spouse who is not an American citizen.
  • There are a couple of ways for deceased Americans to transfer property to their resident alien spouses.
  • One such way includes using up their unified credit. If the decedent was a U.S. citizen or resident, the estate can still qualify for the exemption equivalent ($12,920,000 in 2023) allowing up to this amount to be passed tax-free even to a non-U.S. citizen spouse.
  • Section 2056A allows a marital deduction for property placed into a special trust for the benefit of the non-citizen spouse called the Qualified Domestic Trust (QDOT). This section also requires that there are safeguards to ensure that the trust property (if not used up by the alien survivor) is eventually subjected to the U.S. estate tax.
  • The citizen spouse can establish the QDOT as a living trust or through their will. At the spouse’s death the executor would designate the trust as a QDOT on the federal estate tax return (706).
  • If the citizen spouse does not establish this trust the non-citizen spouse can establish their own QDOT before the decedent’s estate tax return is due.
74
Q

What are the requirements for establishing a QDOT?

A

The requirements for establishing a QDOT are:
* The trustee must be a US citizen or a domestic corporation, or a US bank if trust assets exceed $2 million,
* The trust must retain sufficient assets to cover the non-citizen’s spouse’s estate taxes,
* The trust must be set up as a QTIP trust or an Estate Trust, and
* The trustee must approve all distributions of principal, and withhold estate taxes from principal distributions that are not subject to an ascertainable standard (HEMS). Emergency or hardship distributions are possible and are exempt from transfer tax.

75
Q

What is the disadvantage for QDOTs?

A

A disadvantage to transferring assets into a QDOT is that the assets remaining in the trust at the non-citizen spouse’s death are taxed as if they had been included in the citizen spouse’s estate.
* However, non-citizen spouses can use their applicable credit to offset their own estate tax liability.
* Non-citizen spouses can also become US citizens, making them eligible for the unlimited marital deduction.
* QDOTs can be useful estate planning tools for married couples where one of the partners prefers to keep his/her foreign citizenship.

Withdrawals by a noncitizen surviving spouse from a QDOT established before 2010 will continue to be subject to the deferred estate tax provisions through 2023.

76
Q

Describe Disclaimer Trust

A

A disclaimer trust allows the surviving spouse, or any other beneficiary, the opportunity to do post-mortem (after death) estate planning. In the spousal situation, the disclaimer trust is utilized in smaller estates to avoid visiting with an attorney regularly while providing a mechanism for tax planning by surviving spouse.

The trust is also frequently used when the surviving spouse wishes to have the decedent’s estate take full advantage of the applicable credit available to it or when the survivor does not need property bequeathed to the survivor under the terms of the decedent’s will.

This occurs when the decedent’s estate has overqualified the estate for the marital deduction and has failed to take full advantage of the applicable credit.
* By taking advantage of a disclaimer trust, the surviving spouse disclaims a portion of the assets that would have been received from the grantor’s estate.
* Instead, these assets pass into a disclaimer trust, the income from which can be used to benefit the surviving spouse.

The property that is disclaimed remains in the taxable estate of the decedent and takes full advantage of the applicable credit, so there is usually little if any estate tax liability for the grantor’s estate after 2010.

In effect, the property contained in the disclaimer trust usually passes to someone other than the surviving spouse in order to take advantage of both the applicable credit (the disclaimed portion) and the marital deduction (that portion of the property which is not disclaimed but which instead is enjoyed by the spouse or is that property over which the spouse enjoys postmortem control).

The language for the trust is in the estate planning documents and the surviving spouse may elect to disclaim some or all of the property he or she is entitled to. Once the valid disclaimer is filed, the assets so disclaimed transfer into the disclaimer trust. The election of the disclaimer trust by the surviving spouse is an important postmortem election since it ensures both estates of overall estate tax savings by using the combined estate tax savings provided by the use of both the marital deduction and the applicable credit.

77
Q

Match the term with the correction description.
QTIP
Non-marital or Bypass Trust
Disclaimer Trust
Marital Trust
* Provides the decedent’s spouse with control over the decedent’s property. The amount placed avoids estate tax liability of the first decedent.
* Appropriate when the grantor wishes to leave a life income interest to their spouse as well as, other family members such as children or grandchildren.
* Used when the grantor provides income to a spouse for lifetime, but passes the remainder interest to someone else.
* Used when the surviving spouse wishes to file a qualified disclaimer over a portion of the property passing to the survivor from the grantor’s estate.

A
  • Marital Trust - Provides the decedent’s spouse with control over the decedent’s property. The amount placed avoids estate tax liability of the first decedent.
  • Non-marital or Bypass Trust - Appropriate when the grantor wishes to leave a life income interest to their spouse as well as, other family members such as children or grandchildren.
  • QTIP - Used when the grantor provides income to a spouse for lifetime, but passes the remainder interest to someone else.
  • Disclaimer Trust - Used when the surviving spouse wishes to file a qualified disclaimer over a portion of the property passing to the survivor from the grantor’s estate.
78
Q

What are the 2 specific types of trusts may occur by operation of law?

A

Two specific types of trusts may occur by operation of law:
* A resulting trust
* A constructive trust

These trusts protect the rights of the grantor when all beneficiaries die. They also protect the rights of a beneficiary who has been cheated fraudulently by another individual, depriving the beneficiary of a right to the estate.

79
Q

What is a Resulting Trust?

A

A resulting trust is a trust occurring in favor of the grantor.
* It is used when a trust originally designed for another beneficiary fails because all beneficiaries have died, or when the trust has been declared void.
* The grantor becomes the beneficiary in such a trust arrangement.

80
Q

What is a Standby Trust?

A

A** standby trust is usually structured to take effect when the owner is no longer capable of managing the assets**.
* Frequently, this occurs in situations where the grantor has become disabled or where the grantor has left the country for a period of time (for example, three years leave of absence to develop a business enterprise in a foreign country).
* The standby trust is usually revocable in nature.
* In fact, most standby trusts are revocable inter vivos trusts, in which the grantor is also the trustee and beneficiary.
* The grantor would name a successor trustee to manage his assets if he were to become incapacitated since the grantor would be the trust beneficiary.
* If the trust is not funded when the grantor becomes incapacitated, then the agent of the grantor’s durable power of attorney could transfer assets into the trust for the trustee to manage.
* However, if the trust is already funded, this will avoid delays in managing the grantor’s assets at a critical point in his life.

81
Q

Section 4 - Other Types of Trusts Summary

It can be difficult to quantify the various types of trusts that can be used as part of an estate plan. Just as the tax laws change, so do the types of trusts that are available to meet specific client objectives. The number of estate planning trusts varies and will continue to vary, according to the grantor’s needs, the size of the grantor’s gross estate, the estate tax liability due, and the grantor’s specific identifiable objectives.

In this lesson, we have covered the following:
* Charitable Lead Trust pays a fixed income stream to a qualified charity for years, usually not more than 20. At the expiration of the lead period, the remainder interest passes to one or more non-charitable beneficiaries.
* Charitable Remainder Annuity Trust (CRAT) is a trust designed to pay a fixed amount annually to a non-charitable beneficiary with the remainder going to charity.
* Charitable Remainder Unitrust (CRUT) is designed to pay a variable amount to a non-charitable beneficiary with the remainder to charity. The payment is based on asset performance which is revalued annually.
* Irrevocable Life Insurance Trust: A vehicle for holding life insurance policies which removes them from the grantor and the spouse’s gross estates.
* Use of Life Insurance Trust: Whenever an individual (or a couple) faces a death tax in his or her generation and wishes to provide liquidity for payment of those taxes. Is also used to provide funds for spouses, children, and future generations.

A
  • Marital/Non-marital Trust (also commonly referred to as the “A-B trust” or, more currently, the “A-B-Q” trust) is an arrangement designed to give the surviving spouse the use of the family’s economic wealth, while at the same time minimizing, to the extent possible, the total federal estate tax payable at the deaths of both spouses.
  • The “B” or Bypass Non-marital (family) Trust is designed to receive property that is not allocated to the power of appointment trust, the estate trust, or the QTIP trust. Is generally funded with the applicable credit amount.
  • Qualified Domestic Trust (QDOT) is created to utilize a marital deduction in a US citizen’s estate for property passing to a non-citizen spouse.
  • Disclaimer Trust is utilized to allow the surviving spouse, or any other beneficiary, the opportunity to do post-mortem (after death) estate planning.
  • Totten Trust is not a trust. It is a revocable bank account or money market account that automatically transfers on death (TOD) to the account beneficiary, therefore avoiding probate.
  • Resulting Trust is a trust occurring in favor of the grantor.
  • Standby Trust is usually structured to take effect when the owner can no longer manage the assets.
82
Q

For how many years does a charitable lead trust pay a fixed income stream to a qualified charity?
* 20 years
* 25 years
* 15 years
* 30 years

A

20 years

  • A charitable lead trust pays a fixed income stream to a qualified charity, usually for a period not exceeding 20 years.
83
Q

Match the terms with the correct description.
Estate Trust(c)
Marital Trust
QTIP
Non-marital or Bypass Trust
* It provides the decedent’s spouse with control over the decedent’s property.
* It is used when the grantor leaves a life income interest to his spouse and other family members.
* It is used when the grantor provides income to a spouse for her lifetime, but the remainder interest to some other beneficiary.
* It provides income to the grantor’s spouse only when the trustee deems it proper.

A
  • Marital Trust - It provides the decedent’s spouse with control over the decedent’s property.
  • Non-marital or Bypass Trust - It is used when the grantor leaves a life income interest to his spouse and other family members.
  • QTIP - It is used when the grantor provides income to a spouse for her lifetime, but the remainder interest to some other beneficiary.
  • Estate Trust(c) - It provides income to the grantor’s spouse only when the trustee deems it proper.
84
Q

Section 5 - Income Taxation of Trusts

The IRC and its various sections determine the estate, gift, and income tax treatment of trusts established pursuant to local law. If the specific rules and regulations of the IRC are followed, the transfer of property to a trust will be a completed transfer for estate and gift tax purposes, and the trust will be the owner for income tax purposes as well.

In some cases, financial and estate planners may desire variations of these results. Situations may dictate that the transfer be complete for estate tax purposes, but the income continues to be taxable to the grantor.

A

To ensure that you have a thorough understanding of the income taxation of trusts, the following topics will be covered in this lesson:
* Grantor Trust Rules
* Trusts and Estate Tax Liability
* Gift Tax Implication of Trusts
* Termination Conditions

Upon completion of the lesson, you should be able to:
* Explain the tax implications of trusts,
* Explain tax liabilities of trusts, and
* Explain the conditions under which trusts terminate.

85
Q

Describe Tax Treatment of Trusts

A
  • Income taxation of irrevocable trusts follows certain general rules. Beneficiaries are taxed on the trust income that is or can be distributed to them, while income that is accumulated in the trust is taxed to the trust.
  • This is known as the “sharing” concept.

Trusts are either simple trusts or complex trusts.
* Simple trusts are required to distribute all net income to the beneficiaries in the year the income is earned. Beneficiaries are taxed on the income received at their marginal tax brackets and the trust receives a deduction for the income distributed to the trust beneficiaries. With simple trusts, the trust corpus cannot be distributed and no charitable gifts can be made from the trusts. Simple trusts are separate tax entities and have a $300 personal exemption.

  • If the corpus is distributed to beneficiaries, then the trust becomes a complex trust. Note that capital gains are not considered trust income and are treated as additions to corpus. If the trust distributes capital gains the trust cannot take a deduction for the distribution.
  • Trusts are generally taxed on capital gains retained by the trust, therefore even a simple trust may be subject to income tax.

Trusts that do not require income to be distributed, such as “sprinkling” trusts, are considered complex trusts.
* Complex trusts can accumulate income, make distributions of principal, and make gifts to charities.
* The trust takes deductions for income distributed to beneficiaries but is taxed on income that remains in the trust.
* Therefore, a complex trust is a separate tax-paying entity that has a $100 personal exemption.
* Trust income is taxed to the trust at the highest 37% bracket when trust taxable income exceeds $14,440 in 2023.
* By comparison, individuals are taxed at the 37% bracket when their taxable income exceeds $578,126 in 2023.

86
Q

Describe Distributable Net Income (DNI)

A

The trust beneficiary is taxed on income that is required to be distributed to the beneficiary, regardless of whether the income is actually received. If the income accumulates in the trust and is retained by the trust, the trust pays the income tax on it. If the trust allows Trustee discretion to distribute income, then when the trust passes the income to the beneficiaries, the trust receives a distribution deduction and the beneficiaries must pay the tax on the distribution.
* In effect, the beneficiary is taxed on his or her proportional share of the distributable net income (DNI).

DNI is taxable income prior to applying the personal exemption for trusts and deductions for distributions and after the exclusion of capital gains and losses.

DNI is used in the following ways.
* Provides a limit for the deductions a trust or estate can take for the amounts distributed to beneficiaries.
* Limits the amount of trust income taxable to beneficiaries.
* Determines the types of distributions made to beneficiaries.

DNI ensures that a trust or estate receives a deduction for the amounts distributed to the beneficiary, so the distribution is not taxed twice. For example, a trust earns income of $10,000 and distributes $6,000 to a beneficiary. The DNI is $10,000. The trust would receive a deduction of $6,000 and the trust is taxed on the remaining $4,000.

DNI limits the amount of trust income the beneficiary is required to report. The trust deduction is the lesser of the amount distributed to the beneficiary or the DNI. In the example above, if the trust had distributed $12,000 to the beneficiary the trust would deduct $10,000- the DNI. Under the “income first rule,” $10,000 is considered to be income and the remaining $2,000 is a tax-free distribution of trust corpus. The beneficiary is taxed on $10,000 of the $12,000 received.

DNI ensures that the character of the income the trust receives is distributed in the same manner to the beneficiaries. For example, if the trust receives tax-free income, the income distributed will be tax-free to the beneficiary. Deductions for depreciation and transaction and investment expenses can be deducted by the beneficiary as well.

87
Q

What is the Throwback Rule?

A

An important concept that is frequently employed in the taxation of trusts is the concept of the throwback rule.
This rule is designed to tax the beneficiary of a trust that accumulates income.

The throwback rule applies only in certain situations, however, and applies only where the actual distributions from the trust exceed the amount of the DNI.

88
Q

Describe Grantor Trust Rules

A

A grantor trust is a trust in which the grantor has unexpired interest or residual control over the trust assets. “Control” as used in this definition does not mean the type of control that a trustee has over assets which he administers as a fiduciary. The technical definition of a grantor type trust is any trust where the “grantor” retains any unexpired or reversionary interest. There is a great deal of difference between having an unexpired interest or residual control, and being trustee.

The point of the grantor trust rules is to allow the grantor to be treated as the owner of a trust. The grantor trust rules provide that if the owner of property transferred to a trust retains an economic interest in, or control over, the trust, the owner is treated for tax purposes as the owner of the trust property, and all transactions by the trust are treated as transactions of the owner. A grantor trust for income tax purposes can effectively remove assets from the estate and give the grantor the requisite control so that the income earned within the trust will be taxed back to the Grantor.

89
Q

When does the grantor pay income tax on any income produced by the trust?

A

The grantor pays the income tax on any income produced by the trust if the following conditions occur:
* The grantor receives the income directly.
* The grantor may have the income payable to himself or herself, under the terms of the trust.
* The grantor indicates that the income from the trust is to be paid to the grantor’s spouse.
* The grantor indicates that the income is to accumulate for the benefit of the grantor, or the grantor’s spouse.
* The grantor indicates that the income is to be used to pay life insurance premiums for a policy owned by the grantor or the grantor’s spouse.
* The grantor indicates that the income is to be used to satisfy a legal obligation of support for a family member of the grantor (for example, minor children) or for the grantor’s spouse.
* The grantor has the ability to revoke the trust or alter its terms so that the trust terminates or reverts back to the grantor at any time.
* The grantor retains the power to control the beneficial enjoyment of the trust property, including the power to determine who shall receive the property and the terms under which the property can be enjoyed. (For example, “$20,000 per year to my son, Charles, aged 24, as long as he achieves good grades in his medical studies, but if he does not achieve good grades in his medical studies then the income terminates upon the discretion of the grantor.”)
* The grantor possesses specific administrative powers over the trust. He is allowed to borrow against the trust and use the trust corpus as collateral.

If any of these conditions are present, the grantor will be deemed to be the recipient of the income produced by the trust and will be liable for income tax on any income produced by the trust. If the grantor can substitute assets for 50% of the trust property, then the grantor will be required to report 50% of all income, deductions, and credits on the grantor’s income tax return. If the grantor wishes to avoid being taxed on trust income, the grantor should avoid any and all of the nine situations described.

A trust that is defective for income tax purposes should also be defective for estate tax purposes.
* However, the IRC provisions for income, gift, and estate taxes were enacted at different times.
* Therefore, there is not consistency of treatment, which means that a trust can be effective for estate and gift tax purposes, but defective for income tax purposes.
* This divergence invites planning opportunities, and you will learn more about Intentionally Defective Grantor Trusts (IDGTs) in the Business Planning module.

90
Q

Do assets in Revocable Trusts avoid Estate Tax Liability?

A

When a grantor establishes a revocable living trust, the grantor should realize that the assets in such a trust are still included in the grantor’s gross estate. Because the grantor can terminate the trust at will, or revoke it at any time, the grantor possesses incidents of ownership over the trust and its corpus and undistributed income.

The ability to revoke the trust will cause the inclusion of the corpus and all undistributed income in the gross estate of the grantor. Thus, such a trust should be thought of primarily for its flexibility and not as a saver of estate tax.

91
Q

What can happen if a Revocable changes to a Irrevocable Living Trust?

A

A revocable trust becomes irrevocable upon the earlier of the death of the grantor or when the grantor gives up the right to revoke. Generally, a grantor lacks the power to revoke a trust during incompetency. It is possible to specify that as soon as the client is deemed to be incompetent, the trust becomes irrevocable. This would block a person acting under a durable power of attorney from changing the plan of disposition in the revocable trust.

Irrevocability of a trust may trigger adverse gift tax implications.
* A taxable gift can be made when a client parts with dominion and control over property.
* But immediate gift taxation can be blocked.
* The client could retain a life interest in the trust’s assets and delay taxation on the gift of the remainder interest (the portion going at his death to the ultimate beneficiaries of the trust) by the reservation of a testamentary power of appointment over that remainder interest.

This should work even if the grantor is mentally incompetent since it is assumed that he may regain competency at any time before death.

92
Q

Describe Gift Tax Implications of Trusts

A

In general, a revocable trust is not considered a completed gift because the grantor has not parted with dominion and control over the property. Thus, a revocable trust will not be subject to gift tax liability. However, because the grantor has not parted with ownership of the property, it will be included in the grantor’s gross estate, even if the trust is a life insurance trust.

An irrevocable living trust will normally result in a gift and possibly some gift tax liability.
* The value of the gift is the fair market value of the property at the time it is transferred, not its value at the date of the grantor’s death.
* Thus, if the grantor has property that is highly appreciable in nature, it might be a good idea to transfer the property into the irrevocable living trust in order to avoid having the future appreciation included in the gross estate of the grantor.
* If the trust pays income, it may qualify for the annual exclusion amount on the stream of income being distributed to the beneficiary.

As long as the income is distributed to the beneficiary, or as long as the beneficiary has a demand right of the greater of $5,000 or 5% of the corpus, or can demand the lesser of an annual addition to trust corpus, or the maximum amount allowable under a five and five power (5% of the value of the corpus, or $5,000, whichever is greater) the income produced by such trusts qualifies for the gift tax annual exclusion amount and can reduce the grantor’s gift tax liability.

In trusts where the beneficiary does not have a demand right over a portion of the corpus, or where the income from such a trust must accumulate, the gift tax annual exclusion is not available to offset any gift tax liability the grantor may have to pay.

93
Q

List Termination Conditions

A

There are significant tax implications if a trust is terminated in a fashion other than on purpose. In the case of a constructive trust that is imposed in cases of fraud, the conditions under which a trust can terminate should to be examined. These conditions include:
* Termination of Trust: Most trusts terminate in an orderly fashion, according to the terms of the trust agreement. Thus, the trust will terminate when the stated condition in the trust agreement occurs. For example, a non-marital trust may last as long as the surviving spouse is alive. Upon the surviving spouse’s death, the trust terminates and the remainder interest in the property passes to the children of the decedent and spouse (the income recipient of the non-marital trust). When the specified condition occurs upon which the trust will terminate, the corpus and all undistributed income must be distributed to one or more designated remaindermen. Conversely, all undistributed income and corpus must be distributed to the grantor as a reversionary interest.

  • Trusts that Violate the Rule Against Perpetuities: A trust that is established for a noncharitable beneficiary must be capable of being measured. The trust cannot be perpetual but can only last for a period of time that is measured by the lives of any persons alive at the time the trust is created (plus any life in vitro) plus 21 additional years.
  • Thus, a trust can only last for a time period that does not exceed the life of any person in existence at the time the trust is created, plus a period of 21 years and 9 months (to account for any lives in vitro). If the trust is structured to last longer than this time period, then the trust is in violation of the rule against perpetuities and is regarded as void from its inception. Note that some states have legislatively modified or done away with the rule against perpetuities.
  • Termination by Agreement of the Beneficiaries: If all the beneficiaries are legally and mentally competent, they can agree to terminate the trust and distribute the corpus and undistributed income, provided the material purpose of the trust has first been achieved. In most states, the grantor’s intentions take precedence over the beneficiaries’ wishes. However, if the grantor has a specific purpose for creating the trust and that purpose is yet to be achieved or has not been substantially achieved, then the decision by the beneficiaries to terminate the trust will not be binding. Where the material purpose has already occurred (“to provide financial assistance to my children and grandchildren so that they may attend college”), then the beneficiaries of the trust by agreement may terminate the trust and receive the corpus and undistributed income.
94
Q

List other Termination Conditions

A
  • Illegal Purpose of the Trust: Should the trust or any of its purposes be designed for illegal activities, so that the trustee or any of the beneficiaries must commit an act that is illegal or that would be considered an example of tortious conduct, the trust is illegal and will terminate on the occurrence of the act deemed illegal or tortious. A trust that is designed for the sole purpose of evading the grantor’s creditors is an act of fraud (a tort) and the creditors may reach the assets in trust or the court may declare the trust to be invalid. Subsequently, the trust terminates when the grantor attempts to use it in an attempt to defraud a creditor.
  • Merger of Legal and Equitable Title in the Same Beneficiary: In some states, when the trustee and beneficiary of the trust are the same individual, there may be a merger of legal and equitable title in the individual. When this occurs, the trust may terminate by the operation of law. For a trust to be recognized, there should be vesting of the legal title in the trustee. The equitable title should be in possession of the trust beneficiary. When these titles merge in the same person, there may no longer be a reason to have a trust. Therefore, in some states, when merger occurs, the trust terminates. As long as there are multiple trustees and multiple beneficiaries, the trust will still be operative. Also, if there is one trustee but several beneficiaries, or there are several trustees and one beneficiary, the trust is still an operative instrument. It is when the sole trustee is also the sole beneficiary that the merger doctrine applies and at that point the trust may cease to exist.
  • Termination Due to Impossibility of Purpose: If the purpose for which a trust was created cannot be performed due to impossibility of the purpose, then the trust will terminate if a similarly related purpose cannot be found. For example, if a private trust (noncharitable trust) were created for the sole purpose of providing a stream of income to a disabled beneficiary to provide that beneficiary with living expenses and assistance, but the beneficiary died shortly after the trust was created, the trust would terminate unless the terms of the trust specifically provided either:
  • The trust would continue in the event of the death of the beneficiary, or
  • The trust would continue for the express purpose of providing living expenses and assistance to other disabled persons living within the same metropolitan area.
  • If neither of these provisions existed in the terms of the trust agreement, then the trust could fail because of impossibility of purpose.
95
Q

Section 5 - Income Taxation of Trusts Summary

For tax purposes, trusts are treated as separate entities from the grantor, the trustee, and the beneficiary. The basic question in the income taxation of trusts is, “Who will be taxed on trust income-the trust, the beneficiary, or the grantor?” Generally, the burden of taxation falls on either the trust itself or the beneficiary. But the income of the trust can be taxed to the grantor. For this reason, trusts are categorized for income tax purposes as either “simple” trusts or “complex” trusts. Either type of trust may also be a “grantor” trust.

A trust that insures the payment of federal estate taxes is called a “qualified domestic trust.” A QDT is used when property is left in trust for a non-citizen spouse and the unlimited marital deduction is desired. A function of a QDT is to allow the U.S. Treasury to collect estate taxes. This is not a factor in other types of trusts.

There are significant income tax implications for trusts. The implications for these trusts vary, depending upon the type of trust. Simple trusts distribute all their income in the present year.

A

In this lesson, we have covered the following:
* Trusts: A grantor retained trust, as its name implies, is an irrevocable trust into which the grantor places assets and retains an interest for a fixed period of years. An irrevocable life insurance trust is a vehicle for holding life insurance policies. The marital/non-marital trust (also commonly referred to as the “A-B trust” or “A-B-Q” trust) is an arrangement designed to give the surviving spouse full use of the families economic wealth.
* Grantor Trust Rules establish that a grantor of the trust rather than the trust itself or the trust beneficiary is to be taxed on the income produced by the trust in a number of situations.
* Estate Tax Liability occurs because the ability to revoke a trust causes the inclusion of the corpus and all undistributed income in the gross estate of the grantor.
* Gift Tax Implications: A revocable trust is not to be subject to gift tax liability. However, an irrevocable living trust results in gift tax liability.
* Termination of Trusts occurs when a stated period of time or conditions expire. These conditions include situations where the purpose of the trust has been achieved or if the trust outlives its time or if the beneficiaries agree to terminate the trust. It can also terminate if the trust performs an illegal activity. A trust also terminates if the trustee and beneficiary of the trust are the same individual. When this occurs, the trust may terminate by operation of law because of the merger.

96
Q

The Throwback Rule is designed to: (Select all that apply)
* Tax the beneficiary of a trust that accumulates income.
* Tax that portion of the accumulated income as if the income has been distributed.
* Tax when the distributable net income exceeds the actual distributions from the trust.
* Tax the donor of the trust.

A

Tax the beneficiary of a trust that accumulates income.
Tax that portion of the accumulated income as if the income has been distributed.
* The throwback rule is designed to tax the beneficiary of a trust that accumulates income.
* The throwback rule taxes that portion of the accumulated income as if the income had been distributed.
* The throwback rule applies only in certain situations, however, and applies only where the actual distributions from the trust exceed the amount of the DNI.

97
Q

Which of the following distributions are not subject to the throwback rules: (Select all that apply)
* Gifts or bequests of a specific sum of money or property
* Any amount paid for charity
* Distributions of income that accumulate after the beneficiary’s birth or after the beneficiary reaches the age of 21.
* Distributions that do not exceed the trust’s accounting income.

A

Gifts or bequests of a specific sum of money or property
Distributions that do not exceed the trust’s accounting income.
* Gifts or bequests of a specific sum of money or property and distributions that do not exceed the trust’s accounting income are not subject to throwback rules.
* Distribution of income that accumulates before the beneficiary’s birth and before the beneficiary reaches the age of 21 are not subject to throwback rules.
* A specific amount set aside, paid or used for specific charity alone qualifies for non-imposition of throwback rules.

98
Q

The grantor pays the income tax on any income produced by the trust, provided the following conditions occur. (Select all that apply)
* The grantor indicates that the income is to be used to pay life insurance premiums for a policy owned by the grantor or the grantor’s spouse.
* The grantor’s child, who is the trust beneficiary, receives an income distribution from the trust.
* The grantor may have the income payable to him, under the terms of the trust.
* The grantor has the ability to revoke the trust or alter its terms so that the trust terminates or reverts back to the grantor at any time.

A

The grantor indicates that the income is to be used to pay life insurance premiums for a policy owned by the grantor or the grantor’s spouse.
The grantor may have the income payable to him, under the terms of the trust.
The grantor has the ability to revoke the trust or alter its terms so that the trust terminates or reverts back to the grantor at any time.
* It is necessary that a grantor pay income tax on any income produced by the trust when the grantor indicates that the income is to be used to pay life insurance premiums for a policy owned by the grantor or the grantor’s spouse.If the grantor receives the income, then the grantor pays income tax returns.

99
Q

Module Summary

A trust is a recognized legal entity often used to hold legal title to property for the benefit of one or more persons. The person who creates the trust is the trust creator or grantor. Any person or institution holding legal title to the property is the trustee. Persons who ultimately benefit from the trust are called beneficiaries.

There are three main parties to a trust. The trust creator, also known as the grantor, is the person who creates the trust and transfers assets into it. A trustee may be a person or a financial institution such as a bank or a trust company that holds the legal title to the trust estate. There may be one or more trustees. All trustees are obliged to act in accordance to the terms of the trust for the benefit of trust beneficiaries. As legal titleholder of a trust, a trustee is expected to fulfill certain administrative responsibilities and manage the trust assets. Beneficiaries are the people for whom the trust creator intends to benefit from the trust estate. The rights of the beneficiaries, however, depend on the terms and conditions of the trust. Beneficiaries have the “equitable title” to property held in the trust.

The key concepts to remember are:

A
  • Powers of the Trustee: In order to achieve the objectives of a trust, the trustee is entrusted with certain powers.
  • Kinds of Trusts: There are different kinds of trusts tailored to suit the needs of the grantor or the beneficiaries. These include: the Pour-over Trust, GRAT, GRUT, GRIT, Discretionary Trust, Spendthrift Trust, Sprinkling or Spray Trust, Generation-skipping Trust, UGMA, UTMA, 2503(b) Trust and 2503(c) Trust.
  • Charitable Trusts: Such trusts are popular and effectively save on gift and estate taxes. Different trusts which fall under this category include charitable lead trust, CRAT, CRUT and life insurance trust.
  • A/B TRUSTS: These trusts take advantage of a decedent’s unified credit and marital deduction. These include the Marital Trust, Non-marital or Bypass Trust, Q-TIP, Estate Trust and Disclaimer Trust.
  • Specific Trusts: These occur by operation of law. Examples of such a trust would include a resulting trust and a constructive trust. A resulting trust occurs in favor of the grantor when all beneficiaries have died or when the trust has been declared void. A constructive trust is in favor of a person who is unjustly deprived of property by another individual.
  • Income Taxation of Trusts: The trust and the beneficiaries of the trust are considered as separate entities for tax purposes. The grantor pays income tax only if certain conditions occur.
  • Termination of Trusts: occurs under certain conditions expressed in the trust document or according to state law.
100
Q

A __ ____??____ __ is an irrevocable trust that holds a person’s residence, allowing couples or individuals to live in the house rent-free for a specified period of time.
* Homestead Trust
* GRIT
* GRAT
* QPRT

A

QPRT
* A qualified personal residence trust (QPRT) is an irrevocable trust that holds a person’s residence, allowing couples or individuals to live in the house rent-free for a specified period of time.
* At the end of the term, the home will pass gift-tax free to the trust beneficiaries.

101
Q

A revocable living trust __ ____??____ __.
I. is created by the grantor during their lifetime
II. provides the right tor evoke the trust, change its terms, or regain possession of the property in the trust
* I only
* Both I and II
* II only
* Neither I nor II

A

Both I and II

  • A revocable living trust is one created by the grantor during their lifetime in which the grantor retains the right to revoke the trust, change its terms, or regain possession of the property in the trust.
  • A revocable trust becomes irrevocable when the grantor dies.
102
Q

A sprinkling trust can distribute __ ____??____ __ to beneficiaries.
I. income
II. corpus
* Both I and II
* Neither I nor II
* II only
* I only

A

I only
* A sprinkling trust can distribute income to beneficiaries.
* A spray trust can distribute both income and corpus.

103
Q

By establishing a QDOT, a U.S. citizen spouse transfers property into a trust upon death for benefit of the surviving non-citizen spouse and qualifies the estate for the __ ____??____ __.
* generation-skipping transfer exemption
* annual exclusion amount
* marital deduction
* charitable deduction

A

marital deduction
* Section 2056A allows a marital deduction for property placed into a special trust for the benefit of the non-citizen spouse called the Qualified Domestic Trust (QDOT).
* This section also requires that there are safeguards to ensure that the trust property (if not used up by the alien survivor) is eventually subjected to the U.S. estate tax.

104
Q

Which of the following do spendthrift trusts provide to beneficiaries?
* Distributions for health, education, maintenance, or support.
* Access to trust corpus through an ascertainable standard.
* General power of appointment.
* Creditor protection when established solely for supplemental support.

A

Creditor protection when established solely for supplemental support.
* Spendthrift trusts also provide creditor protection if they are established solely for a beneficiary’s supplemental support, not for their general support.
* This prevents the beneficiary from compelling distributions for support in favor of creditors.

105
Q

The __ ____??____ __ generally manages, distributes, and accumulates income and principal as per the terms of a formal written agreement (called a trust instrument) for the benefit of the beneficiaries.
* executor
* corpus
* trustee
* grantor

A

trustee

  • A trustee is a party to whom property is transferred by the grantor, who receives legal title to the property placed in the trust, and who generally manages, distributes, and accumulates income and principal as per the terms of a formal written agreement (called a trust instrument) for the benefit of the beneficiaries.
106
Q

A __ ____??____ __ trust must distribute all income to a beneficiary on at least an annual basis.
* 2503(b)
* UGMA
* 2503(c)
* UTMA

A

2503(b)

  • A 2503(b) trust can provide a beneficiary with a stream of income during the time in which the beneficiary is a minor.
  • The trust must distribute the income to the minor on an annual or more frequent basis. All or portions of gifts to such trusts will qualify as gifts of present interest for the income beneficiaries, and thus are eligible for the annual gift tax exclusion.
107
Q

A gift to a minor through a Section 2503(c) trust will be considered __ ____??____ __.
* tax-exempt
* a gift of present interest
* a gift of future interest
* pass-through income

A

a gift of present interest

  • A gift to a minor through a Section 2503(c) trust will be considered a gift of a present interest (so the gift will qualify for the annual gift tax exclusion) if the income and principal is available for distribution to or on behalf of the beneficiary at any time prior to the time the beneficiary reaches age 21.
108
Q

Trust property is also known as which of the following?
* Possessions
* Belongings
* Corpus
* Assets

A

Corpus

  • Trust property is often referred to as corpus or principal.
109
Q

Choose the correct characteristic(s) of Uniform Transfers to Minors Act (UTMA) trusts. (Select all that apply)
* Typically terminate at the age of 18.
* Allow for testamentary transfers.
* Only stocks, bonds, and mutual funds can be purchased.
* Flexibility.

A

Allow for testamentary transfers.
Flexibility.
* UTMAs are a flexible type of trust that can be used for the benefit of a minor.
* They allow for testamentary transfers into the minor’s account & terminate at age 21 or 25, depending on the state.
* UTMAs permit the use of various investments, including transfers of real estate, partnership interests, and oil and gas interests.